summaryrefslogtreecommitdiffstats
path: root/inhalt.tex
blob: 2957d64dc8fce6ce540d1eb92d17445ee5dea1ad (plain)
1
2
3
4
5
6
7
8
9
10
11
12
13
14
15
16
17
18
19
20
21
22
23
24
25
26
27
28
29
30
31
32
33
34
35
36
37
38
39
40
41
42
43
44
45
46
47
48
49
50
51
52
53
54
55
56
57
58
59
60
61
62
63
64
65
66
67
68
69
70
71
72
73
74
75
76
77
78
79
80
81
82
83
84
85
86
87
88
89
90
91
92
93
94
95
96
97
98
99
100
101
102
103
104
105
106
107
108
109
110
111
112
113
114
115
116
117
118
119
120
121
122
123
124
125
126
127
128
129
130
131
132
133
134
135
136
137
138
139
140
141
142
143
144
145
146
147
148
149
150
151
152
153
154
155
156
157
158
159
160
161
162
163
164
165
166
167
168
169
170
171
172
173
174
175
176
177
178
179
180
181
182
183
184
185
186
187
188
189
190
191
192
193
194
195
196
197
198
199
200
201
202
203
204
205
206
207
208
209
210
211
212
213
214
215
216
217
218
219
220
221
222
223
224
225
226
227
228
229
230
231
232
233
234
235
236
237
238
239
240
241
242
243
244
245
246
247
248
249
250
251
252
253
254
255
256
257
258
259
260
261
262
263
264
265
266
267
268
269
270
271
272
273
274
275
276
277
278
279
280
281
282
283
284
285
286
287
288
289
290
291
292
293
294
295
296
297
298
299
300
301
302
303
304
305
306
307
308
309
310
311
312
313
314
315
316
317
318
319
320
321
322
323
324
325
326
327
328
329
330
331
332
333
334
335
336
337
338
339
340
341
342
343
344
345
346
347
348
349
350
351
352
353
354
355
356
357
358
359
360
361
362
363
364
365
366
367
368
369
370
371
372
373
374
375
376
377
378
379
380
381
382
383
384
385
386
387
388
389
390
391
392
393
394
395
396
397
398
399
400
401
402
403
404
405
406
407
408
409
410
411
412
413
414
415
416
417
418
419
420
421
422
423
424
425
426
427
428
429
430
431
432
433
434
435
436
437
438
439
440
441
442
443
444
445
446
447
448
449
450
451
452
453
454
455
456
457
458
459
460
461
462
463
464
465
466
467
468
469
470
471
472
473
474
475
476
477
478
479
480
481
482
483
484
485
486
487
488
489
490
491
492
493
494
495
496
497
498
499
500
501
502
503
504
505
506
507
508
509
510
511
512
513
514
515
516
517
518
519
520
521
522
523
524
525
526
527
528
529
530
531
532
533
534
535
536
537
538
539
540
541
542
543
544
545
546
547
548
549
550
551
552
553
554
555
556
557
558
559
560
561
562
563
564
565
566
567
568
569
570
571
572
573
574
575
576
577
578
579
580
581
582
583
584
585
586
587
588
589
590
591
592
593
594
595
596
597
598
599
600
601
602
603
604
605
606
607
608
609
610
611
612
613
614
615
616
617
618
619
620
621
622
623
624
625
626
627
628
629
630
631
632
633
634
635
636
637
638
639
640
641
642
643
644
645
646
647
648
649
650
651
652
653
654
655
656
657
658
659
660
661
662
663
664
665
666
667
668
669
670
671
672
673
674
675
676
677
678
679
680
681
682
683
684
685
686
687
688
689
690
691
692
693
694
695
696
697
698
699
700
701
702
703
704
705
706
707
708
709
710
711
712
713
714
715
716
717
718
719
720
721
722
723
724
725
726
727
728
729
730
731
732
733
734
735
736
737
738
739
740
741
742
743
744
745
746
747
748
749
750
751
752
753
754
755
756
757
758
759
760
761
762
763
764
765
766
767
768
769
770
771
772
773
774
775
776
777
778
779
780
781
782
783
784
785
786
787
788
789
790
791
792
793
794
795
796
797
798
799
800
801
802
803
804
805
806
807
808
809
810
811
812
813
814
815
816
817
818
819
820
821
822
823
824
825
826
827
828
829
830
831
832
833
834
835
836
837
838
839
840
841
842
843
844
845
846
847
848
849
850
851
852
853
854
855
856
857
858
859
860
861
862
863
864
865
866
867
868
869
870
871
872
873
874
875
876
877
878
879
880
881
882
883
884
885
886
887
888
889
890
891
892
893
894
895
896
897
898
899
900
901
902
903
904
905
906
907
908
909
910
911
912
913
914
915
916
917
918
919
920
921
922
923
924
925
926
927
928
929
930
931
932
933
934
935
936
937
938
939
940
941
942
943
944
945
946
947
948
949
950
951
952
953
954
955
956
957
958
959
960
961
962
963
964
965
966
967
968
969
970
971
972
973
974
975
976
977
978
979
980
981
982
983
984
985
986
987
988
989
990
991
992
993
994
995
996
997
998
999
1000
1001
1002
1003
1004
1005
1006
1007
1008
1009
1010
1011
1012
1013
1014
1015
1016
1017
1018
1019
1020
1021
1022
1023
1024
1025
1026
1027
1028
1029
1030
1031
1032
1033
1034
1035
1036
1037
1038
1039
1040
1041
1042
1043
1044
1045
1046
1047
1048
1049
1050
1051
1052
1053
1054
1055
1056
1057
1058
1059
1060
1061
1062
1063
1064
1065
1066
1067
1068
1069
1070
1071
1072
1073
1074
1075
1076
1077
1078
1079
1080
1081
1082
1083
1084
1085
1086
1087
1088
1089
1090
1091
1092
1093
1094
1095
1096
1097
1098
1099
1100
1101
1102
1103
1104
1105
1106
1107
1108
1109
1110
1111
1112
1113
1114
1115
1116
1117
1118
1119
1120
1121
1122
1123
1124
1125
1126
1127
1128
1129
1130
1131
1132
1133
1134
1135
1136
1137
1138
1139
1140
1141
1142
1143
1144
1145
1146
1147
1148
1149
1150
1151
1152
1153
1154
1155
1156
1157
1158
1159
1160
1161
1162
1163
1164
1165
1166
1167
1168
1169
1170
1171
1172
1173
1174
1175
1176
1177
1178
1179
1180
1181
1182
1183
1184
1185
1186
1187
1188
1189
1190
1191
1192
1193
1194
1195
1196
1197
1198
1199
1200
1201
1202
1203
1204
1205
1206
1207
1208
1209
1210
1211
1212
1213
1214
1215
1216
1217
1218
1219
1220
1221
1222
1223
1224
1225
1226
1227
1228
1229
1230
1231
1232
1233
1234
1235
1236
1237
1238
1239
1240
1241
1242
1243
1244
1245
1246
1247
1248
1249
1250
1251
1252
1253
1254
1255
1256
1257
1258
1259
1260
1261
1262
1263
1264
1265
1266
1267
1268
1269
1270
1271
1272
1273
1274
1275
1276
1277
1278
1279
1280
1281
1282
1283
1284
1285
1286
1287
1288
1289
1290
1291
1292
1293
1294
1295
1296
1297
1298
1299
1300
1301
1302
1303
1304
1305
1306
1307
1308
1309
1310
1311
1312
1313
1314
1315
1316
1317
1318
1319
1320
1321
1322
1323
1324
1325
1326
1327
1328
1329
1330
1331
1332
1333
1334
1335
1336
1337
1338
1339
1340
1341
1342
1343
1344
1345
1346
1347
1348
1349
1350
1351
1352
1353
1354
1355
1356
1357
1358
1359
1360
1361
1362
1363
1364
1365
1366
1367
1368
1369
1370
1371
1372
1373
1374
1375
1376
1377
1378
1379
1380
1381
1382
1383
1384
1385
1386
1387
1388
1389
1390
1391
1392
1393
1394
1395
1396
1397
1398
1399
1400
1401
1402
1403
1404
1405
1406
1407
1408
1409
1410
1411
1412
1413
1414
1415
1416
1417
1418
1419
1420
1421
1422
1423
1424
1425
1426
1427
1428
1429
1430
1431
1432
1433
1434
1435
1436
1437
1438
1439
1440
1441
1442
1443
1444
1445
1446
1447
1448
1449
1450
1451
1452
1453
1454
1455
1456
1457
1458
1459
1460
1461
1462
1463
1464
1465
1466
1467
1468
1469
1470
1471
1472
1473
1474
1475
1476
1477
1478
1479
1480
1481
1482
1483
1484
1485
1486
1487
1488
1489
1490
1491
1492
1493
1494
1495
1496
1497
1498
1499
1500
1501
1502
1503
1504
1505
1506
1507
1508
1509
1510
1511
1512
1513
1514
1515
1516
1517
1518
1519
1520
1521
1522
1523
1524
1525
1526
1527
1528
1529
1530
1531
1532
1533
1534
1535
1536
1537
1538
1539
1540
1541
1542
1543
1544
1545
1546
1547
1548
1549
1550
1551
1552
1553
1554
1555
1556
1557
1558
1559
1560
1561
1562
1563
1564
1565
1566
1567
1568
1569
1570
1571
1572
1573
1574
1575
1576
1577
1578
1579
1580
1581
1582
1583
1584
1585
1586
1587
1588
1589
1590
1591
1592
1593
1594
1595
1596
1597
1598
1599
1600
1601
1602
1603
1604
1605
1606
1607
1608
1609
1610
1611
1612
1613
1614
1615
1616
1617
1618
1619
1620
1621
1622
1623
1624
1625
1626
1627
1628
1629
1630
1631
1632
1633
1634
1635
1636
1637
1638
1639
1640
1641
1642
1643
1644
1645
1646
1647
1648
1649
1650
1651
1652
1653
1654
1655
1656
1657
1658
1659
1660
1661
1662
1663
1664
1665
1666
1667
1668
1669
1670
1671
1672
1673
1674
1675
1676
1677
1678
1679
1680
1681
1682
1683
1684
1685
1686
1687
1688
1689
1690
1691
1692
1693
1694
1695
1696
1697
1698
1699
1700
1701
1702
1703
1704
1705
1706
1707
1708
1709
1710
1711
1712
1713
1714
1715
1716
1717
1718
1719
1720
1721
1722
1723
1724
1725
1726
1727
1728
1729
1730
1731
1732
1733
1734
1735
1736
1737
1738
1739
1740
1741
1742
1743
1744
1745
1746
1747
1748
1749
1750
1751
1752
1753
1754
1755
1756
1757
1758
1759
1760
1761
1762
1763
1764
1765
1766
1767
1768
1769
1770
1771
1772
1773
1774
1775
1776
1777
1778
1779
1780
1781
1782
1783
1784
1785
1786
1787
1788
1789
1790
1791
1792
1793
1794
1795
1796
1797
1798
1799
1800
1801
1802
1803
1804
1805
1806
1807
1808
1809
1810
1811
1812
1813
1814
1815
1816
1817
1818
1819
1820
1821
1822
1823
1824
1825
1826
1827
1828
1829
1830
1831
1832
1833
1834
1835
1836
1837
1838
1839
1840
1841
1842
1843
1844
1845
1846
1847
1848
1849
1850
1851
1852
1853
1854
1855
1856
1857
1858
1859
1860
1861
1862
1863
1864
1865
1866
1867
1868
1869
1870
1871
1872
1873
1874
1875
1876
1877
1878
1879
1880
1881
1882
1883
1884
1885
1886
1887
1888
1889
1890
1891
1892
1893
1894
1895
1896
1897
1898
1899
1900
1901
1902
1903
1904
1905
1906
1907
1908
1909
1910
1911
1912
1913
1914
1915
1916
1917
1918
1919
1920
1921
1922
1923
1924
1925
1926
1927
1928
1929
1930
1931
1932
1933
1934
1935
1936
1937
1938
1939
1940
1941
1942
1943
1944
1945
1946
1947
1948
1949
1950
1951
1952
1953
1954
1955
1956
1957
1958
1959
1960
1961
1962
1963
1964
1965
1966
1967
1968
1969
1970
1971
1972
1973
1974
1975
1976
1977
1978
1979
1980
1981
1982
1983
1984
1985
1986
1987
1988
1989
1990
1991
1992
1993
1994
1995
1996
1997
1998
1999
2000
2001
2002
2003
2004
2005
2006
2007
2008
2009
2010
2011
2012
2013
2014
2015
2016
2017
2018
2019
2020
2021
2022
2023
2024
2025
2026
2027
2028
2029
2030
2031
2032
2033
2034
2035
2036
2037
2038
2039
2040
2041
2042
2043
2044
2045
2046
2047
2048
2049
2050
2051
2052
2053
2054
2055
2056
2057
2058
2059
2060
2061
2062
2063
2064
2065
2066
2067
2068
2069
2070
2071
2072
2073
2074
2075
2076
2077
2078
2079
2080
2081
2082
2083
2084
2085
2086
2087
2088
2089
2090
2091
2092
2093
2094
2095
2096
2097
2098
2099
2100
2101
2102
2103
2104
2105
2106
2107
2108
2109
2110
2111
2112
2113
2114
2115
2116
2117
2118
2119
2120
2121
2122
2123
2124
2125
2126
2127
2128
2129
2130
2131
2132
2133
2134
2135
2136
2137
2138
2139
2140
2141
2142
2143
2144
2145
2146
2147
2148
2149
2150
2151
2152
2153
2154
2155
2156
2157
2158
2159
2160
2161
2162
2163
2164
2165
2166
2167
2168
2169
2170
2171
2172
2173
2174
2175
2176
2177
2178
2179
2180
2181
2182
2183
2184
2185
2186
2187
2188
2189
2190
2191
2192
2193
2194
2195
2196
2197
2198
2199
2200
2201
2202
2203
2204
2205
2206
2207
2208
2209
2210
2211
2212
2213
2214
2215
2216
2217
2218
2219
2220
2221
2222
2223
2224
2225
2226
2227
2228
2229
2230
2231
2232
2233
2234
2235
2236
2237
2238
2239
2240
2241
2242
2243
2244
2245
2246
2247
2248
2249
2250
2251
2252
2253
2254
2255
2256
2257
2258
2259
2260
2261
2262
2263
2264
2265
2266
2267
2268
2269
2270
2271
2272
2273
2274
2275
2276
2277
2278
2279
2280
2281
2282
2283
2284
2285
2286
2287
2288
2289
2290
2291
2292
2293
2294
2295
2296
2297
2298
2299
2300
2301
2302
2303
2304
2305
2306
2307
2308
2309
2310
2311
2312
2313
2314
2315
2316
2317
2318
2319
2320
2321
2322
2323
2324
2325
2326
2327
2328
2329
2330
2331
2332
2333
2334
2335
2336
2337
2338
2339
2340
2341
2342
2343
2344
2345
2346
2347
2348
2349
2350
2351
2352
2353
2354
2355
2356
2357
2358
2359
2360
2361
2362
2363
2364
2365
2366
2367
2368
2369
2370
2371
2372
2373
2374
2375
2376
2377
2378
2379
2380
2381
2382
2383
2384
2385
2386
2387
2388
2389
2390
2391
2392
2393
2394
2395
2396
2397
2398
2399
2400
2401
2402
2403
2404
2405
2406
2407
2408
2409
2410
2411
2412
2413
2414
2415
2416
2417
2418
2419
2420
2421
2422
2423
2424
2425
2426
2427
2428
2429
2430
2431
2432
2433
2434
2435
2436
2437
2438
2439
2440
2441
2442
2443
2444
2445
2446
2447
2448
2449
2450
2451
2452
2453
2454
2455
2456
2457
2458
2459
2460
2461
2462
2463
2464
2465
2466
2467
2468
2469
2470
2471
2472
2473
2474
2475
2476
2477
2478
2479
2480
2481
2482
2483
2484
2485
2486
2487
2488
2489
2490
2491
2492
2493
2494
2495
2496
2497
2498
2499
2500
2501
2502
2503
2504
2505
2506
2507
2508
2509
2510
2511
2512
2513
2514
2515
2516
2517
2518
2519
2520
2521
2522
2523
2524
2525
2526
2527
2528
2529
2530
2531
2532
2533
2534
2535
2536
2537
2538
2539
2540
2541
2542
2543
2544
2545
2546
2547
2548
2549
2550
2551
2552
2553
2554
2555
2556
2557
2558
2559
2560
2561
2562
2563
2564
2565
2566
2567
2568
2569
2570
2571
2572
2573
2574
2575
2576
2577
2578
2579
2580
2581
2582
2583
2584
2585
2586
2587
2588
2589
2590
2591
2592
2593
2594
2595
2596
2597
2598
2599
2600
2601
2602
2603
2604
2605
2606
2607
2608
2609
2610
2611
2612
2613
2614
2615
2616
2617
2618
2619
2620
2621
2622
2623
2624
2625
2626
2627
2628
2629
2630
2631
2632
2633
2634
2635
2636
2637
2638
2639
2640
2641
2642
2643
2644
2645
2646
2647
2648
2649
2650
2651
2652
2653
2654
2655
2656
2657
2658
2659
2660
2661
2662
2663
2664
2665
2666
2667
2668
2669
2670
2671
2672
2673
2674
2675
2676
2677
2678
2679
2680
2681
2682
2683
2684
2685
2686
2687
2688
2689
2690
2691
2692
2693
2694
2695
2696
2697
2698
2699
2700
2701
2702
2703
2704
2705
2706
2707
2708
2709
2710
2711
2712
2713
2714
2715
2716
2717
2718
2719
2720
2721
2722
2723
2724
2725
2726
2727
2728
2729
2730
2731
2732
2733
2734
2735
2736
2737
2738
2739
2740
2741
2742
2743
2744
2745
2746
2747
2748
2749
2750
2751
2752
2753
2754
2755
2756
2757
2758
2759
2760
2761
2762
2763
2764
2765
2766
2767
2768
2769
2770
2771
2772
2773
2774
2775
2776
2777
2778
2779
2780
2781
2782
2783
2784
2785
2786
2787
2788
2789
2790
2791
2792
2793
2794
2795
2796
2797
2798
2799
2800
2801
2802
2803
2804
2805
2806
2807
2808
2809
2810
2811
2812
2813
2814
2815
2816
2817
2818
2819
2820
2821
2822
2823
2824
2825
2826
2827
2828
2829
2830
2831
2832
2833
2834
2835
2836
2837
2838
2839
2840
2841
2842
2843
2844
2845
2846
2847
2848
2849
2850
2851
2852
2853
2854
2855
2856
2857
2858
2859
2860
2861
2862
2863
2864
2865
2866
2867
2868
2869
2870
2871
2872
2873
2874
2875
2876
2877
2878
2879
2880
2881
2882
2883
2884
2885
2886
2887
2888
2889
2890
2891
2892
2893
2894
2895
2896
2897
2898
2899
2900
2901
2902
2903
2904
2905
2906
2907
2908
2909
2910
2911
2912
2913
2914
2915
2916
2917
2918
2919
2920
2921
2922
2923
2924
2925
\section*{Motivation} \markboth{}{Motivation}
In der klassischen Analyis haben wir Funktionen im $\K^n$, wobei $\K$ entweder $ℝ$ oder $ℂ$ ist, untersucht.
Dabei war das Betrachten von Eigenschaften wie Konvergenz, Stetigkeit und Differenzierbarkeit sehr nützlich.
Die Funktionalanalysis beschäftigt sich nun mit vergleichbaren Problemen in üblicherweise unendlich"=dimensionalen Funktionenräumen.
Hierfür werden wir versuchen, die aus der klassischen Analysis bekannten Untersuchungsmethoden zu verallgemeinern.
Doch zunächst ein paar Probleme, für deren Lösung man die Funktionalanalysis benötigt.

\begin{problem-nn}
    Ein klassisches Beispiel aus der Variationsrechnung:
    Wir wollen die Funktion
    \[
        f(u) = \int_0^\pi |u'(x)|^2 dx
    \]
    unter den Nebenbedingungungen $u(0) = u(\pi ) = 0$ und $\int_0^\pi |u(x)|^2 dx = 1$ minimieren.
    In der klassischen Analysis haben wir für Minimierungsprobleme mit Nebenbedingungungen Lagrange-Multiplikatoren genutzt.
    Im unendlich"=dimensionalen Fall ist das jedoch nicht so einfach.
    Wir betrachten $f : Y → ℝ$ wie oben, wobei $Y$ eine Teilmenge des unendlich"=dimensionalen Funktionenraums
    \[
        X = \left\{ u ∈ C^1[0,\pi ]: u(0) = u(\pi ) = 0 \right\}
    \]
    ist, die durch
    \[
        Y = \left\{ u ∈ X: \int_0^\pi |u(x)|^2 dx = 1 \right\}
    \]
    gegeben ist.
    Zwar ist $Y$ (in der $\L^2([0,\pi ])$-Metrik) beschränkt und abgeschlossen, jedoch nicht kompakt.
\end{problem-nn}
\begin{problem-nn}
    Sei $\mathcal T = \{ 1, \cos t, \sin t, \cos (2t), \sin (2t), … \} =
    \{\phi_i\}_{i ∈ ℕ}$. Dann ist bekanntlich
    \[
        \langle \phi_i, \phi_j \rangle = ∫_0^{2\pi } φ_i(t) φ_j(t) dt = 2\pi  \delta _{i,j},
    \]
    wobei $\delta _{i,j}$ das Kronecker-Delta bezeichne.
    Also lässt sich durch Normierung ein Orthonormalsystem aus $\mathcal T$ gewinnen.
    Jetzt fragen wir uns, ob sich jede $2\pi $-periodische Funktion $u$ bezüglich eines geeigneten Konvergenzbegriffs in eine Reihe $u = \sum_{i ∈ ℕ} \alpha _i φ_i$ mit $\alpha _i ∈ ℝ$ entwickeln können.
    Bereits bekannt ist, dass das für das entsprechende endlich-dimensionale Problem geht: Sei $T = \{ e_1,…,e_n\}$ die kanonische Standardbasis des $ℝ^n$
    Dann gilt bekanntlich
    \[
        \langle e_i, e_j \rangle_{ℝ^n} = \delta _{i,j}
    \]
    und für jedes $x ∈ ℝ^n$ ist
    \[
        x = \sum_{i=1}^n \alpha _i e_i, \quad \alpha _i = \langle x, e_i \rangle_{ℝ^n}.
    \]
    Wir fragen uns nach den Zusammenhängen zwischen den Problemen im endlich- und unendlich"=dimensionalen.
\end{problem-nn}
\begin{problem-nn}
    Das Biegemoment eines Trägers kann man als Randwertaufgabe (gesucht ist $u: [0,1] → ℝ$, gegeben sind $p,r: [0,1] → ℝ$)
    \[
        u''(t) + p(t) u(t) = r(t), \quad u(0) = u(1) = 0
    \]
    bestimmen. Mit Hilfte der sogenannten Green'schen Funktion lässt sich diese Randwertaufgabe in eine Integralgleichung
    \[
        (T_u)(t) \coloneq ∫_0^1 G(t,s) \big(r(s)-p(s)u(s)\big) ds = u
    \]
    umwandeln. Das heißt, man sucht einen Fixpunkt eines Integraloperators $T$ in einer geeigneten Menge von Funktionen.
\end{problem-nn}

Diese Probleme lassen sich mit der klassischen Analysis nicht mehr behandeln.
In der Funktionalanalysis behandeln wir nun im Wesentlichen „Analysis in $\infty$-dimensionalen Räumen“ (meist Funktionenräume).
Das heißt, wir wollen jetzt anstelle des $\K^n$ allgemeinere Räume betrachten, die jodoch immer noch folgende beide Charakteristika aufweisen:
\begin{enumerate}
\item Die lineare Struktur (das heißt, Elemente lassen sich addieren und mit einem Skalar multiplizieren)
\item Die topologische Struktur (also insbesondere ein Konvergenzbegriff)
\end{enumerate}

Unser Ziel ist es zunächst, die beiden Strukturen zu erarbeiten.

\chapter{Die lineare Struktur}
\section{Der lineare Raum}
Sei im folgenden stets $\K = ℝ$ oder $\K = ℂ$. Zunächst die
\begin{definition}[Vektorraum]
    Sei $\K$ ein Körper. Eine Abelsche Gruppe $(X,+)$ zusammen mit einer Abbildung
    \[
        \cdot : \K × X → X
    \]
    heißt $\K$-Vektorraum, falls für alle $\alpha , β ∈ \K$ und $x, y ∈ X$ gilt:
    \begin{enumerate}[label=(V\arabic*)]
    \item $\alpha  x+y) = \alpha x + βy$
    \item $(\alpha +β)x = \alpha x + βx$
    \item $(\alpha β)x = \alpha (βx)$
    \item $1 \cdot x = x$
    \end{enumerate}
\end{definition}

\begin{bemerkung-nn}
    Je nachdem, ob $\K = ℂ$ oder $\K = ℝ$ gilt, heißt $X$ ein \emph{komplexer} oder ein \emph{reeller} Vektorraum.
\end{bemerkung-nn}

\begin{bemerkung-nn}
    Eine nichtleere Teilmenge $Y ⊂ X$ ist bereits dann ein linearer Raum, falls aus $\alpha , β ∈ \K$, $x, y ∈ Y$ bereits $\alpha x + βy ∈ Y$ folgt, also $Y$ abgeschlossen unter den Vektorraumoperationen ist.
    $Y$ heißt dann \emph{linearer Teilraum} oder auch \emph{linearer Unterraum}.
\end{bemerkung-nn}

\begin{bemerkung-nn}
    Zu jeder Teilmenge $M ⊂ X$ bildet die Menge aller Linearkombinationen von je endlich vieler Elemente einen linearen Teilraum von $X$.
    Dieser heißt die \emph{lineare Hülle} von $M$ oder der \emph{Aufspann} von $M$
    \[
        \lspan M = \left\{ x ∈ X: ∃ l ∈ ℕ, \alpha _1,…,\alpha _l ∈ \K, m_1,…,m_l ∈ M \text{ mit } \sum_{i=1}^l \alpha _i m_i = x \right\}.
    \]
\end{bemerkung-nn}

\begin{bemerkung-nn}
    $M = \{x_\lambda \}_{\lambda\Lambda } ⊂ X$ heißt \emph{Basis} oder \emph{Hamel-Basis} von $X$, falls $M$ \emph{linear unabhängig}, das heißt,
    $0 ∈ X$ lässt sich nur auf triviale Art und Weise als Linearkombination endlich vieler der $x_\lambda $ schreiben, und $\lspan M = X$ ist.
\end{bemerkung-nn}

\begin{bemerkung-nn}
    Besitzt $X$ eine Basis von $n < \infty $ Elementen, dann heißt $n$ die \emph{Dimension} von $X$ und wir schreiben $\dim X = n$.
    Andernfalls heißt $X$ \emph{unendlich-dimensional} ($\dim X = \infty $).
\end{bemerkung-nn}

\begin{bemerkung-nn}
    Seien $X_1, X_2 ⊂ X$ lineare Teilräume. Dann ist
    \[
        X_1 + X_2 \coloneq \left\{ \alpha x_1 + βx_2: \alpha , β ∈ \K, x_1 ∈ X_1, x_2 ∈ X_2 \right\}
    \]
    ebenfalls ein linearer Teilraum.
    Falls $X_1 ∩ X_2 = \{ 0\}$, schreiben wir $X_1 \oplus X_2$ und nennen die Summe \emph{direkt}.
\end{bemerkung-nn}

\begin{bemerkung-nn}
    Sei $Y$ ein linearer Teilraum von $X$. Definiere die Äquivalenzrelation $\sim$ auf $X$ durch
    $x \sim y \Leftrightarrow x - y ∈ Y$.
    Dann wird die Menge der Äquivalenzklassen mit vertreterweiser Addition und Multiplikation auch ein $\K$-Vektorraum.
    Wir schreiben für diesen Vektorraum $X/Y$.
\end{bemerkung-nn}

\section{Beispiele}
\begin{beispiel}
    Der $ℝ^n$ ist ein linearer Raum über dem Körper $ℝ$. Der $ℂ^n$ ist sowohl ein $ℂ$- als auch ein $ℝ$-Vektorraum.
\end{beispiel}

\begin{beispiel}
    Sei $[a,b] ⊂ ℝ$, $a < b$. Dann ist
    \[
      C[a,b] = \{x: [a,b]\K, x \text { ist stetig}\}
    \]
    ein $\K$-Vektorraum mit $\dim C[a,b] = \infty $.
    Zum Beispiel sind die Monome $(t^k)_{k ∈ ℕ}$ ein unendliches linear unabhängiges System, jedoch keine Basis.
    Tatsächlich ist jede Basis dieses Raumes überabzählbar.
\end{beispiel}

\section{Lineare Abbildungen}
\begin{definition}
    Seien $X, Y$ lineare Räume über $\K$. $A: X → Y$ heißt \emph{linear}, falls für alle $x_1, x_2 ∈ X$ und $\alpha , β ∈ \K$ gilt:
    \[
    A(\alpha x_1 + βx_2)  = \alpha A(x_1) + βA(x_2).
    \]
    $A: X → \K$ heißt \emph{lineares Funktional}.
    Für $A$ linear heißt $R(A) = \im A = \{A(x): x ∈ X\}$ der \emph{Bildraum} von $A$ und $N(A) = \ker A = \{ x ∈ X: A(x) = 0\}$ der \emph{Kern} von $A$.
\end{definition}

\begin{bemerkung}
    Sei $A: X → Y$ linear.
    \begin{enumerate}
    \item Sei $M ⊂ X $ ein linearer Unterraum. Dann ist $A(M) ⊂ Y$ wieder ein linearer Unterraum und es gilt $\dim A(M) \le \dim M$ mit Gleichheit bei Injektivität.
    \item Es gilt
        \[
            A \text{ injektiv} \Longleftrightarrow N(A) = \{ 0\}.
        \]
        Allgemeiner ist
        \[
            X/(N(A)) \cong \im A.
        \]
    \item
        Falls $\dim X = \dim Y = n < \infty $, dann ist $A$ genau dann injektiv, wenn $A$ surjektiv ist.
    \item
        $A: X → Y$ ist linear und bijektiv genau dann, wenn es eine lineare Umkehrabbildung $A^{-1}: Y → X$ gibt.
    \item
        Falls so ein $A: X → Y$ linear und bijektiv existiert, nennen wir $X$ und $Y$ \emph{linear isomorph.}
        $A$ heißt dann ein \emph{linearer Isomorphismus}.

        Nur falls $\dim X = \dim Y < \infty $ sind $X$ und $Y$ auch „topologisch“ isomorph.
        In diesem Fall erhält man die Prototypen $ℝ^n$ und $ℂ^n$ für endlich-dimensionale Vektorräume und andere gitbt es nicht (die sie auch als Topologische Räume isomorph sind).
    \end{enumerate}
\end{bemerkung}

\begin{beispiel-nn}
    $X = \{ x: [a,b] → ℝ, x, \dot x, \ddot x \text{ stetig},\; x(a) = \dot x(a) = 0\}$ ist ein linearer Raum.
    Sei $Y = C[a,b]$ und $A: X → Y$ gegeben durch
    \[
        (Ax)(t) \coloneq \ddot x(t) + c_1 (t) \dot x (t) + c_2 (t) x(t), \quad t ∈ [a,b], c_1,c_2 ∈ C[a,b].
    \]
    Dann ist $A$ linear, weil differenzieren linear ist und $A$ ist injektiv:
    Zunächst ist $x = 0$ eine Lösung der linearen Differentialgleichung zweiter Ordnung $Ax = 0$.
    Die Theorie der Differentialgleichungen sagt uns, dass diese Differentialgleichung eine eindeutige Lösung des Anfangswertsproblems ist.

    $A$ ist aber auch surjektiv: Sei $y ∈ Y$ gegeben, dann suchen wir $x ∈ X$ mit $Ax = y$.
    Also wollen wir eine inhomogene Differentialgleichung zweiter Ordnung lösen.
    Auch diese ist nach der Theorie von gewöhnlichen Differentialgleichungen eindeutig lösbar.

    Also ist $A$ bijektiv, das heißt, es gibt eine lineare Abbildung $A^{-1}: Y → X$.
    Diese Inverse ist in der Regel schlecht anzugeben.
    Einen einfacheren Spezialfall dazu wird in der Übung behandelt.
\end{beispiel-nn}

\begin{beispiel-nn}
    Sei $X = Y = C[a,b]$, $A: X → X$ gegeben durch
    \[
        (Ax)(t) \coloneq ∫_a^b k(s,t) x(s) ds, \quad t ∈ [a,b],
    \]
    wobei $k : [a,b] × [a,b] → ℝ$ stetig und gegeben ist.
    Dann ist $A$ linear, da das Integral linear ist.
    Auch ist, wenn $\lambda  ∈ ℝ$ ein Parameter ist, die Abbildung
    \[
        (A_\lambda x)(t) \coloneq \lambda x(t) - (Ax)t), \quad t ∈ [a,b]
    \]
    linear.
    Die Probleme $Ax = y$ (bei gegebenem $y ∈ Y$ und gesuchtem $x ∈ X$) oder $A_\lambda  x = 0$ (gesucht ist $\lambda  ∈ ℝ$ und eine nichttriviale Lösung $x ∈ X \setminus \{ 0\}$)
    heißen Integralgleichungen erster und zweiter Ordnung.
\end{beispiel-nn}

\begin{beispiel-nn}
    Sei $X = C[a,b]$, $A : X → ℝ$ mit
    \[
        Ax = x(t_0),
    \]
    wobei $t_0 ∈ [a,b]$ fest gewählt sei.
    Eine andere lineare Abbildung $A: X → ℝ$ ist gegeben durch
    \[
        Ax = ∫_a^b x(t) dt
    \]
    Dann sind beide Abbildungen $A$ linear und nicht injektiv, aber surjektiv.
\end{beispiel-nn}

\begin{beispiel-nn}
    Sei $X = \ell^2$, $A: X → X$. Für $x = (ξ_n)_{n ∈ ℕ}$ sei
    \[
        Ax = (0,ξ_1, ξ_2, \dots) ∈ \ell^2.
    \]
    $A$ heißt (Rechts-)Shiftoperator und ist linear und injektiv, jedoch nicht surjektiv.
    Solche Abbildungen gibt es für $\dim X = \dim Y < \infty $ nicht.
\end{beispiel-nn}

\section{Duale Räume}
$A: X → \K$ sei ein lineares Funktional, $X$ ein linearer Raum. Wir verwenden ein neues Symbol (statt $A$)
\[
    x': X → \K = \begin{cases} ℝ \\ ℂ \end{cases} \text{ linear}.
\]
Wir schreiben nun
\[
    x'(x) =: \langle  x, x' \rangle = \langle  x, x' \rangle_{X × X^f}\K.
\]
Wir setzen
\[
    X^f \coloneq \left\{ x': x' \text{ ist lineares Funktional auf } X \right\}.
\]
Hierbei sollte man nicht $x'$ nicht mit der Ableitung von $x$ verwechseln.
Auch ist $\langle  -, - \rangle_{X × X^f}$ kein Skalarprodukt.

Der Raum $X^f$ wird auf natürlicher Weise zum linearen Raum mit
\[
    (\alpha x_1' + βx_2')(x) \coloneq \alpha x_1'(x) + βx_2'(x), \quad x ∈ X, x_1', x_2' ∈ X^f, \alpha , β ∈ \K.
\]
So ist
\[
    \langle -,- \rangle_{X×X^f}: X × X^f → \K
\]
bilinear.
\begin{definition}
    $X^f$ heißt der \emph{algebraische Dualraum} zu $X$.
    $X^{ff} \coloneq (X^f)^f$ heißt der \emph{biduale Raum} zu $X$.
\end{definition}

\begin{beispiel-nn}
    $X^{ff}$ liefert die kanonische Abbildung
    \[
        J: X → X^{ff}, \; x ↦ J(x) = x''
    \]
    mit
    \[
        \langle x', x'' \rangle \coloneq \langle x. x' \rangle \quad ∀x' ∈ X^f.
    \]
    Damit ist $x'': X^f → \K$ linear wohldefiniert.
\end{beispiel-nn}

\begin{definition}
    Der lineare Raum $X$ heißt \emph{algebraisch reflexiv}, falls $J$ bijektiv ist (und damit $X$ linear isomorph zu $X^{ff}$) ist.
\end{definition}

\begin{bemerkung}
    $X$ ist genau dann algebraisch reflexiv, wenn $\dim X < \infty $ ist.

    Im Fall $\dim X < \infty $ lässt sich leicht eine duale Basis angeben:
    Sei dazu $M \coloneq \{x_1,…,x_n\}$ eine Basis von $X$. Dann wird durch
    \[
        \langle  x_i, x_k' \rangle \coloneq \delta _{i,k}
    \]
    und linearer Fortsetzung die Menge $ M \coloneq \{x_1',…,x_n'\} ⊂ X^f$ erklärt.
    Dann ist $M'$ eine Basis von $X'$, die die \emph{duale Basis} von $M$ genannt wird.
    Tatsächlich ist $X^f$ im Falle $\dim X = \infty $ wesentlich größer.
    Man wählt deshalb eine (neue) Defintion des Dualraums:
\end{bemerkung}

\begin{definition}[Dualraum]
    Zu einem linearen Raum $X$ ist
    \[
        X' \coloneq \left\{ x' : X → \K, x' \text{ linear und stetig} \right\} ⊂ X^f
    \]
    der Dualraum von $X$.
\end{definition}
Um Allerdings von Stetigkeit reden zu können, müssen wir zunächst \emph{Topologien} einführen.

\chapter{Topologie}
\section{Topologische Räume}
\begin{definition}
    Sei $X$ eine Menge und $\mathcal T ⊂ \Pot X$ eine Menge von Teilmengen von $X$.
    $\mathcal T$ heißt eine \emph{Topologie} auf $X$, falls $\mathcal T$ unter endlichen Durchschnitten und beliebigen Vereinigungen abgeschlossen ist.
    Insbesondere muss $\mathcal T$ $\emptyset$ als leere Vereinigung und $X$ als leeren Schnitt enthalten.
    $(X,\T)$ heißt dann \emph{topologischer Raum}. Die Elemente von $\T$ heißen \emph{offene Mengen}
\end{definition}
\begin{beispiele}
    \begin{enumerate}[label=(\alph*)]
        \item
              Für alle Mengen $X$ ist $\T = \{ ∅, X\}$ eine Topologie auf $X$, die sogenannte \emph{indiskrete Topologie}, \emph{gröbste Topologie} oder auch \emph{Klumpentopologie}.
        \item
              Für alle Mengen $X$ ist $\T = \Pot X$ eine Topologie, die sogenannte \emph{diskrete Topologie} oder \emph{feinste Topologie} auf $X$.
        \item
            In Analysis I wird eine Menge $U ⊂ ℝ$ für offen erklärt, wenn es zu jedem $x ∈ U$ ein $\epsilon  > 0$ gibt, so dass für alle $ y ∈ ℝ$ mit $|x - y| < \epsilon $ auch $y ∈ U$ gilt.
            Aus der Analysis ist bekannt, dass die so definierten offenen Mengen den Axiomen genügen.
            Diese Topologie $\Tnat$ wird \emph{natürliche Topologie} genannt.
        \item
              Sei $X$ eine beliebige Menge. Die \emph{cofinite Topologie} auf
              $X$ wird definiert als
              \[
                  \Tcof = \{ Y ⊂ X: Y = ∅\; \text{oder}\; \complement_X Y\, \text{ist endlich}\}
              \]
        \item
              Der \emph{Sierpinski-Raum} ist die Menge $\{0,1\}$ versehen mit der Topologie $\{ ∅, \{0\}, \{0,1\}\}$.
    \end{enumerate}
\end{beispiele}

\begin{definition}
    Sei $M ⊂ X$.
    \begin{enumerate}
    \item
        $M$ heißt \emph{abgeschlossen}, wenn $X \setminus  M$ offen ist.
    \item
        $U ⊂ X$ heißt \emph{Umgebung von $A$}, wenn es eine offene Menge $V$ gibt mit $A ⊂ V ⊂ U$. Wir setzen
        \[
            \U_A \coloneq \U_A (\T) \coloneq \{ U ⊂ X : U\; \text{Umgebung von $A$}\}.
        \]
        $\U_A$ heißt \emph{Umgebungssystem} oder \emph{Umgebungsfilter} von $A ⊂ X$.
        Für $x ∈ X$ setzen wir $\U_x \coloneq \U_{\{x\}}$. $x$ heißt dann \emph{innerer Punkt} von $U$ für alle $U ∈ \U_x$.
    \item
        $x ∈ X$ heißt \emph{Häufungspunkt} von $M$, falls jede Umgebung von $x_0$ ein $y ∈ M$ enthält mit $y \ne x$.k
    \item
        Das \emph{Innere von M} ist
        \[
            M^\circ \coloneq \bigcup \left\{ U ∈ \T: U ⊂ M \right\}
        \]
        die größte offene Menge, die in $M$ enthalten ist.
    \item
        Der \emph{Abschluss von} M ist
        \[
            \cl M \coloneq \bigcap \left\{ U ⊂ M: U \text{ abgeschlossen} \right\}
        \]
        die kleinste abgeschlossene Menge, die $M$ enthält.
    \item
        $M$ heißt \emph{kompakt}, falls jede offene Überdeckung von $M$ eine endliche Teilüberdeckung besitzt.
    \item
        $M$ heißt \emph{dicht}, falls $\cl M = X$.
    \item
        $M$ heißt \emph{nirgends dicht}, falls $(\cl M)^\circ = \emptyset$.
    \end{enumerate}
\end{definition}
\begin{bemerkung}
    \begin{enumerate}
    \item $M^\circ ⊂ M ⊂ \cl  M$.
    \item
        $M^\circ$ ist die Menge der inneren Punkte von $M$.
    \item
        $M$ ist genau dann abgeschlossen, wenn $M = \cl M$.
    \end{enumerate}
\end{bemerkung}


%%%%% VORLESUNG VOM DONNERSTAG, 19. OKTOBER FEHLT
\begin{definition}[Hausdorff-Raum]
	Sei $(X,\T)$ eine topologischer Raum.
	Für alle $x,y \in X$ mit $x \neq y$ 
	existieren $U \in \U_x, V \in \U_x$ mit $U \cap V = \emptyset$.
	Dann heißt $(X,\T)$ Hausdorff-Raum bzw. genügt dem Trennungsaxiom.
\end{definition}

\begin{definition}[Konvergenz]
	Eine Folge $\{x_{n}\}_{n \in \N} \subset X$ heißt konvergent gegen $x_{0} \in X$,
	falls zu jeder Umgebung $U \in \U_{x_{0}}$ ein $n_{0} \in \N$ existiert, 
	sodass $x_{n} \in U$ für alle $n \geq n_{0}$.
\end{definition}
\begin{bemerkung}
	Man überlegt sich leicht, dass der Grenzwert $x_{0}$ in der Regel nicht eindeutig ist.
	Bsp: In $\T=\{X,\emptyset\}$ konvergiert jede Folge gegen jeden Punkt.
	Ist $(X,\T)$ jedoch ein Hausdorff-Raum, so ist jeder Grenzwert eindeutig.
\end{bemerkung}
\begin{beweis}
	Seien $x_{0} \neq x'_{0}$ Grenzwerte von $(x_{n})_{n \in \N} \subset X$.
	Dann existieren disjunkte Umgebungen $U,U' ∈ \U_{x_0}$.
	Weiterhin gibt es ein $n_{0} \in \N$, so dass $x_{n} \in U$ für alle $n \geq n_{0}$
	und $n_0' \in \N$, so dass $x_{n} \in U'$ für alle $n \geq n_0'$.
	Also gilt $x_{\max\{n_0,n'_0\}} \in U \cap U'$
	Das ist ein Widerspruch zur Disjunktheit der Umgebungen.
\end{beweis}

\begin{definition}[Häufungspunkt]
	$x_{0} \in X$ heißt Häufungspunkt von $\{x_{n}\}_{n \in \N} \subset X$,
	falls zu jeder Umgebung $U \in \U_{x_{0}}$ und für alle $k \in \N$ 
	ein $n \geq k \in \N$ existiert, sodass $x_{n} \in U$.
\end{definition}
\begin{beispiel}
	$\{x_{n}\}_{n \in \N} \subset \R$ mit natürlicher Topologie.
	$x_{n}=(-1)^n$ hat zwei HP $\pm 1$
	Achtung: $M=\{x_{n}:n \in \N\}=\{-1,1\}$ hat als Menge keine HP.
\end{beispiel}
\begin{bemerkung}
	Für die indiskrete Topologie ist jeder Punkt in X HP jeder Folge.
\end{bemerkung}

\begin{definition}[Stetigkeit]
	$f : (X,\T_{X}) \rightarrow (Y,\T_{Y})$ heißt stetig, falls
	für alle $V \in \T_{Y}$ gilt, dass $f^{-1}(V) \in \T_{X}$.
\end{definition}
\begin{bemerkung}
	$f$ ist stetig $\Longleftrightarrow$ $f$ ist stetig in jedem Punkt
\end{bemerkung}

\begin{definition}[Homöomorphismus]
	Ist $f : (X,\T_{X}) \rightarrow (Y,\T_{Y})$ bijektiv und stetig,
	und $f^{-1} : (Y,\T_{Y}) \rightarrow (X,\T_{X})$ auch stetig,
	dann heißt $f$ Homöomorphismus.
	$X$ und $Y$ heißen homöomorph, falls so ein Homöomorphismus existiert.
\end{definition}

\begin{definition}[Basis von Topologien und Umgebungen]
	\begin{enumerate}
	\item
	Eine Familie $B \subset \T$ heißt Basis der Topologie in $(X,\T)$, falls
	$T={\cup M: M \subset B}$.
	\item
	Eine Familie $B \subset \U_{x}$ von $x \in X$ heißt Umgebungsbasis des Punktes $x$,
	falls für alle $U \in \T, x \in U$ existiert ein $V \in B$ mit $x \in V \in U$. 
	\end{enumerate}
\end{definition}
\begin{beispiel}
	Für die natürliche Topologie auf $\R^n$ ist eine Basis der Topologie gegeben durch
	${B_{\eps}(x): x \in X, \eps > 0}$ 
	mit den offenen Kugeln $B_{\eps}(x)={y \in R^n : \norm{x-y}<\eps}$.
	Sei $x \in \R^n$ fest. 
	Dann ist ${B_{1/n}(x):n \in \N}$ eine abzählbare Umgebungsbasis von x
\end{beispiel}

\begin{definition}[Relativtopologie oder Spurtopologie]
	$M \subset \T$ eines topologischen Raumes $(X,\T)$ lässt sich in natürlicher Weise
	zu einem topologischen Raum machen, nämlich mit $\T' \coloneq {M \cap V : V \in \T}$.
\end{definition}
\begin{bemerkung}
	$M = M \cap X \in \T'$ da $X \in \T$, d.h. $M$ ist offen in der Spurtopologie.
	Achtung: $M$ muss nicht offen in $X$ sein.
\end{bemerkung}

\begin{definition}
	Seien zwei Topologien $\T_{1},\T_{2}$ auf X gegeben. 
	Wir sagen $\T_{1}$ ist feiner als $\T_{2}$, falls $\T_{1} \supset \T_{2}$.
	Wir sagen $\T_{1}$ ist gröber als $\T_{2}$, falls $\T_{1} \subset \T_{2}$.
	Wir sagen die Topologien sind gleich, falls $\T_{1}=\T_{2}$.
\end{definition}
\begin{bemerkung}
	Sei $\T_{1}$ feiner als $\T_{2}$.
	Die feinere Topologie $\T_{1}$ enthält mehr offene Mengen, 
	und damit zu jedem Grenzwert $x_{0}$ weniger konvergte Folgen.
	
	Man zeigt leicht: 
	$\T_{1}$ ist feiner als $\T_{2}$ $\Longleftrightarrow$ 
	Für alle $x \in X$ gilt: Seien $B_{1} \subset T_{1},B_{2} \subset T_{2}$ Umgebungsbasen von $x$, 
	dann gilt für alle $U \in B_{1}$, dass ein $V \in B_{2}$ existiert mit $V \subset U$.
\end{bemerkung}

\begin{beispiel}
	Folgende Topolgien auf $\R^n$ sind gleich.
	$\T_{1}$ sei die Topologie, die durch die Kugeln 
	$B_{\eps}(x)={y \in R^n : \norm{x-y}<\eps}$ erzeugt wird.
	$\T_{2}$ sei die Topologie, die durch die Quader 
	$B_{\eps}(x)={y \in R^n : \max_{1 \leq i \leq n} |y_{i}-x_{i}|<\eps}$ erzeugt wird.
\end{beispiel}

\begin{definition}[Produkttopologie]
	Seien $(X,\T_{X}),(Y,\T_{Y})$ topologische Räume.
	Dann sit die Familie von Mengen
	$\{U_{X} \times U_{Y} : U_{X} \in \T_{X}, U_{Y} \in \T_{Y} \} \subset 2^{X \times Y}$
	eine Basis der Topologie $\T_{X \times Y}$ im kartesischen Produkt $X \times Y$.
	Bemerkung: Es genügt auch wenn $U_{X},U_{Y}$ über Basen von $\T_{X},\T_{Y}$ genommen werden.
\end{definition}

\section{Metrische Räume}
\begin{lemma}[Eigenschaften metrischer Räume]
    Sei $(X,d)$ ein metrischer Raum.
    \begin{enumerate}
    \item Jeder Punkt $x ∈ X$ besitzt eine abzählbare Umgebungsbasis
        \[
            \{ B_{1/n} (x), n ∈ ℕ\}.
        \]

    \item
        Es gilt
        \[
            \lim_{n \to \infty } x_n = x \; \Longleftrightarrow \; \lim_{n→\infty } d(x,x_n) = 0.
        \]
    \item
        Es ist $x_0 ∈ M$ genau dann ein innerer Punkt von $M ⊂ X$, wenn ein $\epsilon  > 0$ existiert mit $B_\epsilon (x_0) ⊂ M$.
    \item
        $M$ ist nirgends dicht in $X$ genau dann, wenn es zu jeder Kugel
$B_\epsilon (x_0)$ mit $x_0 ∈ X, \epsilon > 0$ eine Kugel $B_\delta (x_1) ⊂
B_\epsilon (x_0)$ mit $B_\delta(x_1) ∩ M = \emptyset$ gibt.
    \item
        Seien $(X,d_X)$ und $(Y,d_Y)$ metrische Räume.
        Dann ist auch $(X×Y,d_{X×Y})$ ein metrischer Raum vermöge der Metrik
        \[
            d_{X×Y}((x_1,y_1),(x_2,y_2)) \coloneq \max\{d_x(x_1,x_2),d_y(y_1,y_2)\}
        \]
        oder auch mit 
        \[
            d_{X×Y}((x_1,y_1),(x_2,y_2)) \coloneq \sqrt{d_x^2(x_1,x_2)+d_y^2(y_1,y_2)}.
        \]
        Tatsächlich induzieren diese beiden Metriken die gleiche Topologie (nämlich die Produkttopologie)
    \item
        Homöomorphismen $f: X → Y$ (für metrische Räume $X, Y$), die die Metrik respektieren, das heißt
        \[
            d_X(x_1,x_2) = d_Y(f(x_1),f(x_2)) \quad ∀x_1, x_2 ∈ X
        \]
        heißen \emph{Isometrien}.
    \item
        Ein metrischer Raum muss im allgemeinen keine lineare Struktur haben.
        Man betrachte hierzu die Menge $X \coloneq \{1,2,3,4,5,6\}$ mit der diskreten Metrik.
        Diese kann keine Vektorraumstruktur haben, da $|X| = 6$ keine Primzahlpotenz ist.
    \end{enumerate}
\end{lemma}
\begin{proof}
Der Beweis wird aufgrund seiner Trivialität den Lesern zur Übung überlassen, da er wirklich nur Einsetzen der Definitionen ist.
\end{proof}

Nun ein paar Charakterisierungen von kompakten Mengen in metrischen Räumen.
\begin{satz}
    Im metrischen Raum $(X,d)$ sind äquivalent:
    \begin{enumerate}
    \item
        $K ⊂ X$ ist kompakt (überdeckungskompakt)
    \item
        Jede Folge in $K$ besitzt mindestens einen  Häufungspunkt in $K$ (abzählbar kompakt)
    \item
        Jede Folge in $K$ besitzt eine konvergente Teilfolge mit Grenzwert in $K$ (folgenkompakt)
    \end{enumerate}
\end{satz}

\begin{bemerkung}
    Der Satz gilt so im allgemeinen Hausdorff-Raum \emph{nicht}.
    Für „$(b) \Rightarrow (a)$“ benötigt man zusätzlich das zweite Abzählbarkeitsaxiom, also die Existenz einer abzählbaren Basis der Topologie.
    Für „$(b) \Rightarrow (c)$“ benötigt  man das erste Abzählbarkeitsaxiom, also die Existenz von abzählbaren Umgebungsbasen für jeden Punkt.
\end{bemerkung}

\section{Vollständigkeit in metrischen Räumen und der Satz von Baire}
\begin{definition}
    Eine Folge $(x_n)_{n ∈ ℕ} ⊂ X$ in $(X,d)$ heißt \emph{Cauchy-Folge}, falls zu jedem $\epsilon  > 0$ ein $N = N(\epsilon )$ existiert mit $d(x_m,x_n) < \epsilon $ für alle $n,m \ge N$.
\end{definition}

\begin{lemma}
    Jede Konvergente Folge $(X_n)_{n ∈ ℕ} ⊂ X$ ist auch eine Cauchy-Folge.
\end{lemma}

\begin{definition}
    Der metrische Raum $(X,d)$ heißt \emph{vollständig}, falls jede Cauchy-Folge in $(X,d)$ konvergiert.
\end{definition}

Nicht jeder metrische Raum braucht vollständig zu sein (man betrachte hierfür z.B. $ℚ$ und die Folge der Partialsummen der Dezimalbruchentwicklung von $\sqrt 2$),
jedoch lässt sich jeder metrische Raum zu einem vollständigen Erweitern.

\begin{satz}
    Jeder metrische Raum $(X,d)$ lässt sich in einen bis auf Isometrie eindeutig bestimmten kleinsten vollständigen metrischen Raum $(\tilde X, \tilde d)$ einbetten.
    Dieser Raum $(\tilde X, \tilde d)$ heißt die Vervollständigung von $(X,d)$.
\end{satz}
\begin{proof}
    Zwei Cauchyfolgen $(x_n)_{n ∈ ℕ}$ und $(y_n)_{n ∈ ℕ}$ seien äquivalent, falls $d(x_n,y_n) \xrightarrow[n → \infty ]{} 0$.
    Hierdurch ist eine Äquivalenzrelation definiiert. Sei $[(x_n)_{n ∈ ℕ}]$ die vom Repräsententaten $(x_n)_{n ∈ ℕ}$ erzeugte Klasse. Man setzt
    \[
        \tilde X \coloneq \{ [ (x_n)_{n ∈ ℕ}] : (x_n)_{n ∈ ℕ} \text{ ist Cauchy-Folge in }(X,d)\}
    \]
    und
    \[
        \tilde d([(x_n)_{n ∈ ℕ}],[(y_n)_{n ∈ ℕ}]) \coloneq \lim_{n → \infty } d(x_n,y_n).
    \]
    Dann ist $(d(x_n,y_n))_{n ∈ ℕ}$ eine Cauchy-Folge in $ℝ$, da
    \[
        |d(x_n,x_m) - d(y_m,y_m) | \le \underbrace{d(x_n,x_m)}_{0} + \underbrace{d(y_n,y_m)}_{0}.
    \]
    Da $ℝ$ bekanntlich vollständig ist, existiert somit der Grenzwert.
    Ferner ist $\tilde d$ Repräsentatenunabhängig, also wohldefiniert:
    Seien $(\tilde x_n)$ und $(\tilde y_n)$ andere Repräsentaten. Dann ist
    \[
        d(x_n,y_n) \le \underbrace{d(x_n,\tilde x_n)}_{0} + d(\tilde x_n,\tilde y_n) + \underbrace{d(\tilde y_n, y_n)}_{0}.
    \]
    Die umgekehrte Ungleichung ergibt sich aus Vertauschung der Rollen. Man rechnet leicht nach, dass $(\tilde X, \tilde d)$ ein vollständiger Raum ist.
    Wir können $(X,d)$ durch die entsprechenden konstanten Folgen isometrisch in $\tilde X$ einbetten.
\end{proof}

\begin{bemerkung-nn}
    Wendet man diese Technik auf $ℚ$ mit der natürlichen Metrik an, dann erhält man $(ℝ,d)$ als vollständige Hülle.
    Man beachte jedoch, dass dies nicht für die Konstruktion von $ℝ$ ausreicht, da hier schon die Existenz von $ℝ$ verwenden wird -- Aber das funktioniert größtenteils analog.
\end{bemerkung-nn}


\begin{satz}[Schachtelsatz]\label{schachtelsatz}
    Sei $(X,d)$ ein vollständiger metrischer Raum und seien
    $(x_n)_{n * ℕ} ⊂ X$ und $(r_n)_{n ∈ ℕ} ⊂ (0,\infty ) $ Folgen mit der Eigenschaft
    \begin{enumerate}
    \item $\cl B_{r_{n+1}}(x_{n+1}) ⊂ B_{r_n} (x_n)$
    \item $\lim_{n \to \infty } r_n = 0$.
    \end{enumerate}
    Dann gibt es genau ein $x_0 ∈ X$ mit $x_0 ∈ \bigcap_{n ∈ ℕ} \cl B_{r_n} (x_n)$.
\end{satz}
\begin{proof}

    Für $p ∈ ℕ$ beliebig gilt
    \[
        \cl B_{r_{n+p}} (x_{n+p}) ⊂ \cl B_{r_n} (x_n).
    \]
    Also
    \[
        d(x_{n+p},x_n) \le r_n \xrightarrow[n → \infty ]{} 0.
    \]
    Damit ist $(x_n){n ∈ ℕ}$ eine Cauchyfolge und damit konvergiert gegen ein $x_0 ∈ X$.
    Außerdem gilt
    \[
        d(x_p,x_n) \le \underbrace{d(x_0, x_{n+p})}_{0 (p → \infty )} + \underbrace{d(x_{n+p},x_n)}_{ \le r_n}.
    \]
    Damit folgt für $p → \infty $
    \[
        d(x_0, x_n) \le r_n \quad ∀ n ∈ ℕ
    \]
    also $x_0 ∈ \bigcap_{n ∈ ℕ} \cl B_{r_n}(x_n)$.
    Für die Eindeutigkeit sei $\tilde x_0$ ebenfalls in $\bigcap_{n ∈ ℕ} \cl B_{r_n}(x_n)$.
    Dann folgt
    \[
        d(x_0,\tilde x_0) \le \underbrace{d(x_0,x_n)}_{\le r_n} + \underbrace{d(x_n, \tilde x_0)}_{\le r_n} \le 2r_n \xrightarrow[n → \infty ]{} 0.
    \]
    Doch damit war bereits $x_0 = \tilde x_0$.
\end{proof}

\begin{definition}
    Eine Teilmenge $M$ eines metrischen Raumes $(X,d)$ heißt \emph{von erster Kategorie} oder  \emph{mager}, falls sie
     die Vereinigung abzählbar vieler in $X$ nirgends dichter Mengen ist. Andernfalls heißt $M$ \emph{von zweiter Kategorie}.
\end{definition}

Der folgende Satz wird beim Beweis mehrerer fundamentaler Sätze benötigt, z.B beim Prinzip der gleichmäßigen Beschränktheit oder dem Open-Mapping-Theorem.


\begin{satz}[Baire]\label{baire}
    Jede nichtleere offene Menge eines vollständigen metrischen Raumes $(X,d)$ ist von zweiter Kategorie (insbesondere $X$ selbst)
\end{satz}
\begin{proof}
    Sei  $ \emptyset \ne M ⊂ X$ offen. Wir nehmen umgekehrt an, $M$ wäre von erster Kategorie, das heißt
    \[
        M ⊂ \bigcup_{n ∈ ℕ} M_n
    \]
    mit $M_n ⊂ X$ nirgends dicht. Wähle $x_0 ∈ M$. Da $M$ offen ist, gibt es ein $r = r_0 > 0$ mit $B_{r_0}(x_0) ⊂ M$.
    Da $M_1$ nirgends dicht ist, gibt es $r_1 > 0$ und $x_1 ∈ X$ mit
    \[
        B_{r_1}(x_1) ⊂ B_{r_0/2} (x_0)
    \]
    und $B_{r_1}(x_1) ∩ M_1 = \emptyset$.
    Analog finden wir, da $M_2$ nirgends dicht ist, $r_2 > 0$ und $x_2 ∈ X$ mit
    \[
        B_{r_2}(x_2) ⊂ B_{r_1/2} (x_1)
    \]
    und $B_{r_2}(x_2) ∩ M_2 = \emptyset$.
    Durch Fortsetzen dieses Schemas finden wir eine Folge $(x_n)_{n ∈ ℕ} ⊂ X$ und Radien $(r_n)_{n ∈ ℕ} ⊂ (0,\infty )$ mit $r_n \le r/2^n \xrightarrow[n → \infty ]{} 0$.
    Damit sind alle Voraussetzungen von \cref{schachtelsatz} erfüllt. Folglich existiert genau ein
    \[
        \tilde x ∈ \bigcap_{n ∈ ℕ} B_{r_n} (x_n) ⊂ B_r(x_0) ⊂ M.
    \]
    Aber $\tilde x \not\in M_n$ für alle $n ∈ ℕ$ Folglich ist auch $\tilde x$ nicht in $\bigcup_{n ∈ ℕ} M_n = M$. Das ist ein Widerspruch. Also ist $M$ von zweiter Kategorie.
\end{proof}

% \begin{satz}[Satz von Baire]\label{44-baire}
%     Sei $(X,\T)$ ein vollständig metrisierbarer oder lokalkompakter Hausdorff\hyp{}Raum
%     \begin{enumerate}
%     \item
%         Sei $(U_n)_{n ∈ ℕ}$ eine Folge offener, dichter Teilmengen von $X$.
%         Dann ist auch $\bigcap_{n ∈ ℕ} U_n ⊂ X$ dicht.
%     \item
%         Sei $(A_n)_{n ∈ ℕ}$ eine Folge nirgends dichter Teilmengen. Dann ist
%         $\bigcup_{n ∈ ℕ} A_n \ne X$.
%     \item
%         Sei $(A_n)_{n ∈ ℕ}$ eine Folge abgeschlossener Teilmengen mit
%         $\bigcup_{n ∈ ℕ} A_n = X$. Dann gilt für mindestens ein $n ∈ ℕ$, dass $A_n^\circ
%         \ne \emptyset$.
%     \end{enumerate}
% \end{satz}
% \begin{proof}
%     \begin{enumerate}
%     \item
%         Sei $W ⊂ X$ offen und nichtleer. Zu zeigen: $\bigcap_{n ∈ ℕ} U_n ∩ W \ne
%         \emptyset$. Sei zunächst $X$ vollständig metrisierbar durch die Metrik
%         $d$. Da $U_1 ∩ W$ offen und nichtleer nach Annahme gibt es $x_1 ∈ U ∩ W$
%         und $0 < r_1 < 1$ mit $B_{r_1}(x_1) ⊂ U_1 ∩ W$. Wir wählen nun induktiv
%         Punkte $x_n ∈ X$ und Zahlen $0 < r_n < 1$ (für $n \ge 2$) mit folgenden
%         Eigenschaften:
%         \begin{enumerate}[label=(\roman*)]
%         \item
%             $0 < r_n < \frac 1 n$
%         \item
%             $\cl{B_{r_n}(x_n)} ⊂ U_n ∩ B_{r_{n-1}} (x_{n-1})$
%         \end{enumerate}
%         Dazu beachte man, dass $U_n ∩ B_{r_{n-1}}(x_{n-1})$ nichtleer und offen
%         ist, also existiert $x_n$ und $\frac 1 n > \epsilon  > 0$ mit $B_\epsilon (x_n) ⊂ U_n ∩
%         B_{r_{n-1}} (x_{n-1})$ und $r_n = \frac \epsilon  2$ ist wie gewünscht. Für $m
%         \ge n$ impliziert (ii), dass $x_m ∈ B_{r_n}(x_n)$ und aus (i) folgt,
%         dass die Folge $(x_n)_{n ∈ ℕ}$ damit eine Cauchyfolge ist. Damit
%         konvergiert $(x_n)_{n ∈ ℕ}$ gegen ein $x ∈X$. Sei nun $N ∈ ℕ$ und $m >
%         N$. Dann folgt aus $x_m ∈ B_{r_N}(x_N)$, dass
%         \begin{align*}
%           x &= \lim_{m → \infty } x_m ∈ \cl{B_{r_N}(x_n)} ⊂ U_N ∩ B_{r_{N-1}}(x_{N-1}) \\
%           & ⊂ U_N ∩ B_{r_1}(x_1) ⊂ U_N ∩ W,
%           \end{align*}
%         also $x ∈ \bigcap_{n ∈ ℕ} U_N ∩ W$.

%         Sei Nun $X$ lokalkompakt. Da $U_1 ∩ W$ offen und nichtleer ist, gibt es
%         $x ∈ U_1 ∩ W$, und es ist $U_1 ∩ W ∈ \U_x$. Wähle $B_1 ∈ \U_x$ kompakt
%         mit $B_1 ⊂ U_1 ∩ W$. Wir konstruieren nun sukzessive kompakte Mengen
%         $B_k$ (zu $k \ge 2$) mit folgenden Eigenschaften:
%         \begin{enumerate}[label=(\roman*)]
%         \item
%             $B_k ⊂ B_{k-1}$
%         \item
%             $\emptyset \ne B_k^\circ ⊂ B_k ⊂ U_k$.
%         \end{enumerate}
%         Ist $B_{k-1}$ gegeben, so ist wegen $B_{k-1}^\circ \ne \emptyset$ und
%         der Dichtheit von $U_k$ der Schnitt $B_{k-1}^\circ ∩ U_k$ offen und
%         nichtleer. Es gibt also ein $x ∈ B_{k-1}^\circ ∩ U_k$ und damit auch
%         eine kompakte $x$-Umgebung $B_k ⊂ U_k ∩ B_{k-1}$.
%         Die Familie $(B_k)_{k ∈ ℕ}$ nichtleerer Teilmengen der kompakten Menge
%         $B_1$ ist absteigend, besitzt also die endliche Durschnittseigenschaft.
%         Da $X$ hausdorffsch und die $B_k$ kompakt sind, ist zudem jedes $B_k$
%         abgeschlossen, somit folgt 
%         \[
%             \emptyset \ne \bigcap_{k ∈ ℕ} B_k ⊂ \bigcap _{k ∈ ℕ}U_k
%         \]
%         sowie
%         \[
%             \bigcap_{k ∈ ℕ} B_k ⊂ B_1 ⊂ W.
%         \]
%         Insgesamt also
%         \[
%             \emptyset \ne \bigcap_{k ∈ ℕ} B_k ⊂ \bigcap_{k ∈ ℕ} U_k ∩ W.
%         \]
%     \item
%         Für $n ∈ ℕ$ Sei $U_n = \complement_X \cl{A_n}$. Dann ist $U_n$ offen und
%         dicht. Mit Teil (a) folgt, dass auch $\bigcap_{n ∈ ℕ U_n }$ dicht, und
%         somit insbesondere nicht leer, ist. Also ist $\bigcup_{n ∈ ℕ} A_n
%         ⊂ \bigcup_{n ∈ ℕ} \cl{A_n} = (\bigcap_{n ∈ ℕ} U_n)^\complement \ne X$.
%     \item
%         Das ist eine direkte Konsequenz aus (b).
%     \end{enumerate}
% \end{proof}


\chapter{Topologische lineare Räume}
Erklärtes Ziel dieses Kapitels wird sein, die beiden Strukturen aus den vorherigen beiden Kapiteln, also die Topologie und den linearen Raum zusammenzuführen.
\begin{definition}
    Ein linearer Raum $X$ über dem Körper $\K$ mit Topologie $\T$ heißt \emph{topologischer linearer Raum}, falls die Vektorraumoperationen ($+ : X×X → X$ und $\cdot: \K×X → X$) stetig sind.
\end{definition}

\begin{bemerkung-nn}
    Stetigkeit der Vektorraumoperationen sollte als minimales Kompatibilitätskriterium der beiden Strukturen gefordert werden.
    Tatsächlich ist es im Allgemeinen gar nicht erfüllt. Erst im normierten Raum bekommt man diese Stetigkeit geschenkt.
\end{bemerkung-nn}

\section{Normierte Räume}
\begin{definition}
    Sei $X$ ein linearer Raum über $\K$. Die Abbildung $\norm\cdot: X → [0,\infty )$
    heißt \emph{Norm} auf $X$, falls für alle $x, y ∈ X, \alpha  ∈ K$ gilt:
    \begin{enumerate}
    \item $\norm x = 0 \Longleftrightarrow x = 0$
    \item
        $\norm{\alpha x} = |\alpha | \norm x$
    \item
        $\norm{x+y} \le \norm x + \norm y$
    \end{enumerate}
    $(X,\norm\cdot)$ heißt dann \emph{normierter Raum}.
\end{definition}

\begin{bemerkung}
    Durch $d(x,y) \coloneq \norm{x-y}$ wird ein normierter Raum auch ein metrischer, also insbesondere auch ein topologischer Raum.
    Diese induzierte Topologie auf $(X, \norm\cdot)$ heißt \emph{Normtopologie}.

    Ohne die lineare Struktur macht der normierte Raum gar keinen Sinn, da für die Definition einiger der Normaxiome die Vektorraumoperationen verwendet werden.
\end{bemerkung}

\begin{beispiele}
    \begin{enumerate}
    \item 
        Betrachte den $ℝ^n$ mit $\norm x _{p} \coloneq \left( \sum_{i=1}^n |x_i|^p \right)^{1/p}$ mit $1 \le p < \infty $ ist ein normierter Raum,
        genauso wie mit $\norm{x}_{\infty } \coloneq \max_{1 \le i \le n} |x_i|$.
        Insbesondere gibt es im $ℝ^n$ überabzählbar viele verschiedene Normen.
        Wir werden jedoch später sehen, dass diese Normen alle die gleiche Topologie erzeugen.
    \item
        Der Raum aller stetigen Funktionen auf einem kompaktem Intervall $C[a,b]$ mit $\norm{x}_{\infty } \coloneq \max_{t ∈ [a,b]} |x(t)|$ ist ein normierter Raum.
        Außerdem wird durch
        \[
            \norm x \coloneq ∫_a^b |x(t)| dt
        \]
        ebenfalls eine Norm definiert.
    \item
        Sei $\Omega ⊂ ℝ^n$ offen und beschränkt. Dann wird $C(\cl{\Omega})$ mit
        \[
            \norm{x}_{\infty } \coloneq \max_{t ∈ \cl \Omega} |x(t)|
        \]
        auch zu einem normierten Raum.
    \item
        $L^p(\Omega) = \L^p(\Omega)/\mathcal N$, wobei $\mathcal N = \{ f: \Omega\R, f(t) = 0 \text{ fast überall}\}$ ist mit 
        \[
            \norm x \coloneq \left(∫_{\Omega} |x(t)|^p dt \right)^{1/p}
        \]
        ein normierter Raum, wobei $1 \le p < \infty $.
    \item
        $\ell^p$ mit
        \[
            \norm x _{p} \coloneq \left( \sum_{i=1}^n |x_i|^p \right)^{1/p}
        \]
        ist ebenfalls ein normierter Raum, wobei $1 \le p < \infty $.
    \end{enumerate}
\end{beispiele}

\begin{lemma}
    Sei $(X, \norm\cdot)$ ein normierter Raum. Dann sind die Abbildungen $+$, $\cdot$ und $\norm\cdot$ stetig.
\end{lemma}
\begin{proof}
    Für beliebige Folgen $(x_n)_{n ∈ ℕ},(y_n)_{n ∈ ℕ} ⊂ X, (\alpha _n)_{n ∈ ℕ}$ mit $\lim x_n = x$, $\lim y_n = y$, $\lim \alpha _n = \alpha $ gelten
    \[
        \norm{(x_n + y_n) - (x+y)} \le \norm{x-x_n} + \norm{y -y_n}
    \]
    sowie
    \[
        \norm{\alpha _nx_n - \alpha x} \le |\alpha _n| \norm{x_n-x} + \norm{x} |\alpha _n - \alpha |
    \]
    und
    \[
        |\norm{x_n} - \norm{x}| \le \norm{x_n - x}
    \]
    nach der umgekehrten Dreiecksungleichung.
    Folglich sind die zu betrachtenden Abbildungen alle folgenstetig, und, da metrische Räume stets dem ersten Abzähhlbarkeitsaxiom genügen, auch stetig.
\end{proof}

\begin{korollar}
    Jeder normierte Raum versehen mit der Normtopologie ist ein topologischer linearer Raum.
    Deshalb ist auch keine Unterscheidung zwischen normierten Räumen und normierten linearen Räumen nötig.
\end{korollar}

\section{Topologische lineare Räume}
\begin{bemerkung-nn}
    Hierbei sei stetis die Topologie von $X×X$ die Produktopologie, bei den Körpern $\K = \begin{cases} ℝ \\ ℂ \end{cases}$ die übliche Topologie.
    Wir schreiben im Folgenden für Mengen $M, M_1, M_2 ⊂ X$ und $\alpha\K$ nun
    \[
        M_1 + M_2 \coloneq s(M_1,M_2) \coloneq \{x+y : x ∈ M_1, y ∈ M_2\},
    \]
    \[
        A \cdot M \coloneq m(A,M) \coloneq \{ \alpha x: \alpha  ∈ A, x ∈ M\}.
    \]
\end{bemerkung-nn}

\begin{lemma}
    Hat der topologische Raum $(X,\T)$ auch eine lineare Struktur, so sind äquivalent:
    \begin{enumerate}
    \item Die Addition $s$ ist stetig.
    \item
        Für beliebiges $x, y ∈ X$ gilt: Zu jeder Umgebung $O_{x+y} ∈ \T$ existieren Umgebungen $O_x ∈ \T$ von $x$ und $O_y ∈ \T$ von $y$ mit $O_x + O_y ⊂ O_{x+y}$
    \end{enumerate}
\end{lemma}
\begin{proof}
    $s$ ist stetig in $(x,y)$ genau dann, wenn zu jeder Umgebung $O_{x,y}\T_X$
    von $(x,y)$ existiert eine Umgebung $U ⊂ \T_{X×X}$ von $(x,y)$ mit $s(U) ⊂ O_{x+y}$.
    Nach Definition der Produkttopologie existieren dann Umgebungen $O_x ∈ \U_x$ und $O_y ∈ \U_y$ mit $O_x × O_y ⊂ U$.
    Damit ist
    \[
        O_x + O_y = s(O_x, O_y) = s(O_x × O_y) ⊂ s(U) ⊂ O_{x+y}.
    \]
    Analog zeigt man die entsprechende Aussage für die skalare Multiplikation:
\end{proof}
\begin{lemma}
    Hat der topologische Raum $(X,\T)$ auch eine lineare Struktur, so sind äquivalent:
    \begin{enumerate}
    \item Die Addition $m$ ist stetig.
    \item
        Für beliebiges $\alpha\K, x ∈ X$ gilt: Zu jeder Umgebung $O_{\alpha x} ∈ \T$ existieren Umgebungen $O_x ∈ \T$ von $x$ und $O_\alpha  ∈ \T$ von $y$ mit $O_\alpha  × O_x ⊂ O_{\alpha x}$.
    \end{enumerate}
\end{lemma}

Betrachtet man insbesondere die Stetigkeit am Punke $\alpha =0$ und $x ∈ X$ beliebig, dann gilt also:
Für jede Umgebung $O_0 ∈ \U_0 ⊂ X$ existiert eine Umgebung $O_x ∈ \U_x$ und ein $r > 0$, so dass
\[
    ∀β: |β| <r: βO_x ⊂ O_0.
\]
Unmittelbar daraus erhalten wir folgendes Korollar:
\begin{korollar}
    Im topologischen Raum $(X,\T)$ gilt für $x ∈ X$ beliebig und $(β_n)_{n ∈ ℕ} ⊂ ℝ$
    \[
       β_n \xrightarrow[n → \infty ]{} 0 \implies β_nx \xrightarrow[n → \infty ]{} 0.
    \]
\end{korollar}

\begin{definition-nn}
    \begin{enumerate}
    \item 
        Zu $x_0 ∈ X$ fest definieren wir den Translationsoperator
        \[
            T_{x_0} \coloneq X → X, x ↦ x + x_0.
        \]
    \item
        Zu $\alpha _0 ∈ \K^*$ fest definieren wir den Multiplikationsoperator
        \[
            M_{\alpha _0} \coloneq X → X, x ↦ \alpha _0\cdot x.
        \]
    \end{enumerate}
\end{definition-nn}

\begin{lemma}
    Die Translationsoperatoren und Multiplikationsoperatoren sind Homöomorphismen.
\end{lemma}
\begin{proof}
    Das ist klar.
\end{proof}

\begin{korollar}[Invarianzprinzip]
    Im topologischen linearen Raum $(X,\T)$ ist die Topologie bereits durch die offenen Umgebungen von $0 ∈ X$ bestimmt. Alle anderen offenen Mengen entstehen durch Translation.
\end{korollar}
\begin{proof}
    Das ist klar.
\end{proof}

\section{Metrische lineare Räume und Quasi-normierte Räume}
\begin{definition}
    Eine Metrik $d: X × X → ℝ$ auf einem linearen Raum $X$ heißt \emph{translationsinvariant}, falls gilt:
    \[
        ∀x,y,z ∈ X: d(x,y) = d(x+z, y+z),
    \]
    oder äquivalent dazu:
    \[
        ∀x,y ∈ X: d(x,y) = d(x-y, 0).
k    \]
\end{definition}
\begin{bemerkung-nn}
    Ohne lineare Struktur macht das gar keinen Sinn!
\end{bemerkung-nn}

\begin{definition}
    Ein metrischer Raum $(X,d)$ mit linearer Struktur und translationsinvarianter Mertik $d$ heißt \emph{metrischer linearer Raum}, falls
    die Vektorraumoperationen stetig sind (in der von der Metrik induzierten Topologie).
\end{definition}


\begin{lemma}
    Im metrischen Raum $(X,d)$ mit linearer Struktur und translationsinvarianter metrik, dann ist die Addition immer stetig.
\end{lemma}
\begin{proof}
    Es genügt, da in metrischen Räumen Folgenstetigkeit und Stetigkeit äquivalent sind, zu zeigen, dass $\lim d(x_n + y_n, x + y ) = 0$, sofern $\lim d(x_n,x) = 0$  und $\lim d(y_n,y) = 0$.
    Dazu ist
    \[
        d(x_n+y_n,x+y) \le d(x_n+y_n) + d(x+y_n, x+y) = d(x_n,x) +d(y_n,y) \xrightarrow[n → \infty ]{} 0.
    \]
\end{proof}

\begin{beispiel-nn}
    Sei $X = C(a,b)$ mit der Metrik
    \[
        d(x,y) \coloneq \min\{ 1, \sup_{t ∈ (a,b)} |x(t)-y(t)|\}.
    \]
    Dann ist $d$ eine translationsinvariante Metrik, aber $X$ ist kein linearer Raum, da die Skalarmultiplikation nicht stetig ist.
\end{beispiel-nn}
Für die Stetigkeit der skalaren Multiplikation im Punkt $(\alpha ,x) ∈ \K × X$ hat man (nach dem $\epsilon -\delta -Kriterium$)
\[\epsilon  > 0\delta  > 0 ∃ r> 0 ∀β ∈ \K ∀y ∈ X: 
    \begin{rcases}
        |β - \alpha | < r \\
        d(x,y) < \delta 
    \end{rcases}
    \implies d(βy,\alpha x) < \epsilon 
\]


\begin{lemma}
    \label{lemma-metrischer-linearer-raum-charak}
    Sei $(X,d)$ ein metrischer Raum mit linearer Struktur und mit einer translationinvarianten Metrik.
    Dann ist $X$ mit der von $d$ erzeugten Topologie ein \emph{metrischer linearer Raum} genau dann, wenn für alle $\alpha  ∈ \K, x ∈ X$ und beliebige Nullfolgen $(x_n)_{n ∈ ℕ} ⊂ X, (\alpha _n)_{n ∈ ℕ)}\K$ gilt
    \begin{gather*}
        \alpha x_n \xrightarrow[n → \infty ]{} 0 \\
        \alpha x_n \xrightarrow[n → \infty ]{} 0 \\
        \alpha _nx_n \xrightarrow[n → \infty ]{} 0
    \end{gather*}
\end{lemma}
\begin{proof}
    „$⇒$”: Skalare Multiplikation ist im metrischen linearen Raum stetig, also folgen die Aussagen sofort.

    „$⇐$“: Wegen der Äquivalenz von Stetigkeit und Folgenstetigkeit ist zu zeigen
    \[
        \begin{rcases}
            \alpha _n \xrightarrow[n → \infty ]{} \alpha\K \\
            x_n \xrightarrow[n → \infty ]{} x ∈ X
        \end{rcases}
    \implies \alpha _n x_n \xrightarrow[n → \infty ]{} \alpha x.
    \]

    Sei dazu $z_n \coloneq x_n - x ∈ X$, $γ_n \coloneq \alpha _n - \alpha\K$. Dann ist
    \[
        γ_n z_n + γ_n x + \alpha  z_n = (\alpha _n - \alpha )(x_n-x) + (\alpha _n-\alpha ) x + \alpha (x_n-x)
        = \alpha _n x_n - \alpha ×.
    \]
    Somit ist
    \begin{align*}
        d(\alpha _nx_n,\alpha x) &= d(\alpha nx_n - \alpha x,0) = d(γ_nz_n + γnx + \alpha z_n, 0) \\
        &\le \underbrace{d(γ_nz_n,0)}_{0} + \underbrace{d(γ_nx, 0)}_{0} +
\underbrace{d(\alpha z_n, 0)}_{0} \xrightarrow{n → \infty } 0.
    \end{align*}
    Da die Addition ohnehin immer stetig ist, sind wir  fertig.
\end{proof}


\begin{definition}
    Eine Abbildung $|\cdot|: X → [0,\infty )$ heißt \emph{Quasi-Norm} auf dem Linearen
    Raum $X$, falls gilt:
    \begin{enumerate}[label=(Q\arabic*)]
    \item
        $|x| \ge 0$ für alle $x ∈ X$ und $|x| = 0$ genau dann, wenn $x = 0$.
    \item
        $|-x| = |x|$ für alle $x ∈ X$
    \item
        $|x+y| \le |x| + |y|$ für alle $x,y ∈ X$
    \item
        $|\alpha x_n| \xrightarrow[n → \infty ]{} 0$ für $\alpha\K$, falls $|x_n| → 0$
    \item
        $|\alpha _nx| \xrightarrow[n → \infty ]{} 0$ für $x ∈ X$, falls $|\alpha _n| → 0$
    \item
        $|\alpha _nx_n| \xrightarrow[n → \infty ]{} 0$ falls $|x_n| → 0$ und $|\alpha _nx_n| → 0$
    \end{enumerate}
    $(X,|\cdot|)$ heißt dann \emph{quasi-normierter} Raum.
\end{definition}

\begin{bemerkung}
    Jeder normierte Raum ist auch ein quasi-normierter Raum.
\end{bemerkung}

\begin{satz}
    \begin{enumerate}
    \item
        Ist $|\cdot|$ eine Quasi-Norm auf $X$, so wird durch $d(x,y) \coloneq |x-y|$ eine translationsinvariante Metrik definiert, welche $X$ zu einem metrischen linearen Raum macht.
    \item
        Ist $(X,d)$ ein metrischer linearer Raum mit translationsinvarianter Metrik $d$, so ist
        $(X,|\cdot|)$ mit $|x| \coloneq d(x,0)$ ein quasi-normierter Raum.
    \end{enumerate}
\end{satz}
\begin{proof}
    Das folgt direkt aus den Axiomen und \cref{lemma-metrischer-linearer-raum-charak}.
\end{proof}


Speziell für die Anwendung sehr wichtige metrische lineare Räume werden von Semi-Normen erzeugt.

\begin{definition}
    Sei $X$ ein linearer Raum.
    Eine Abbildung $p: X → ℝ$ heißt \emph{Semi-Norm} oder \emph{Halbnorm}, falls folgendes gilt:
    \begin{enumerate}[label=(S\arabic*)]
    \item
        $∀x ∈ X: p(x) \ge 0$
    \item
        $∀ x ∈ X, \alpha  ∈ \K: p(\alpha x) = |\alpha | p(x)$
    \item
        $∀ x, y ∈ X: p(x+y) \le p(x) + p(y)$
    \end{enumerate}
    $(X,p)$ heißt dann \emph{semi-normierter} Raum.
\end{definition}

\begin{beispiel-nn}
    $\L^p(\Omega)$ ist ein semi-normierter Raum.
\end{beispiel-nn}

\begin{bemerkung-nn}
    Jeeder semi-normierte Raum $(X,p)$ erzeugt einen normierten Raum $(X/N,p)$, wobei $N = \{ x ∈ X: p(x) = 0\}$ ein linearer Unterraum ist.
\end{bemerkung-nn}

\begin{satz}
    \label{satz-abzaehlbares-prod-seminormierter-raeume}
    Es seien $p_n: X → ℝ, n ∈ ℕ$ abzählbar viele Semi-Normen auf einem linearen Raum mit der Eigenschaft
    \begin{equation}
        p_n(x) = 0  \text{ für alle n ∈ ℕ } \implies x = 0. \label{eq:seminorm-folge-blub}
    \end{equation}
    Dann ist
    \[
        d(x,y) \coloneq \sum_{n = 1}^\infty  2^{-n} \frac{p_n(x-y)}{1+p_n(x-y)}
    \]
    eine translationsinvariante Metrik auf $X$, welche $X$ zum metrischen linearen Raum macht.
\end{satz}

\begin{bemerkung}
    $p_n: X → ℝ$ sind auf $(X,d)$ stetig. Das folgt aus (für $x_i → x_0$ in $X$)
    \[
        |p_n(x_i) - p_n(x_0)| \le p_n(x_i-x_0) \xrightarrow{} 0
    \]
    und einer Übungsaufgabe.
\end{bemerkung}

\begin{satz}
    \label{satz-umgebungsbasis-produkt-von-seminorm}
    Sei $(X,d)$ der in \cref{satz-abzaehlbares-prod-seminormierter-raeume} gegebene metrische lineare Raum (mit der von der Metrik erzeugten Topologie).
    Dann bilden die Mengen ($\epsilon _n > 0$)
    \[
        U (p_n,\epsilon _n) \coloneq \bigcup B^{p_n}_{\epsilon _n}(0)
        = \{ x ∈ X: p_n(x) < \epsilon _n\}
    \]
    und deren endliche Durchschnitte eine Umgebungsbasis von $0 ∈ X$
\end{satz}

\begin{bemerkung-nn}
    Nach dem Invarianzprinzip ist damit durch $\bigcup B^{p_n}_{\epsilon _n}$ die ganze Topologie bestimmt.
    Mit anderen Worten: Die Topologie welche über die Metrik bestimmt ist, ist dieselbe wie die, welche von den
    $U(p_n,\epsilon _n)$ und endlichen Schnitten davon erzeugt wird.
\end{bemerkung-nn}
\begin{proof}[\cref{satz-umgebungsbasis-produkt-von-seminorm}]
    Zunächst ist $U (p_n,\epsilon _n) ∈ \T$:
    Sei $n ∈ ℕ$  und $\epsilon _n > 0$ fest und $y ∈ U(p_n,\epsilon _n)$ beliebig gegeben.
    Dann ist $p_n(y) < \epsilon _n$. Dann wähle $ρ = ρ(y) > 0$, so dass $p_n(y) + ρ < \epsilon _n$.
    Dann gilt für $r \coloneq 2^{-n} \frac{ρ}{1} > 0$:
    \[
        x ∈ B_r(y) \implies p_n(x+r) < ρ.
    \]
    Dazu ist 
    \[
        \frac{p_n(x-y)}{1+p_n(x-y)} \le 2^n \underbrace{d(x,y)}_{< r} < 2^n r = \frac{ρ}{1},
    \]
    also $p_n(x-y) < ρ$. Mit diesem $r$ gilt $B_r(y) ⊂ U(p_n,\epsilon _n)$:
    Sei $x ∈ B_r(y)$. Dann gilt
    \[
        p_n(x) \le \underbrace{p_n(x-y)}_{< ρ} + p_n(y) < p_n(y) + ρ = \epsilon _n
    \]
    wie gewünscht.
    

    Sei $ B_r(0), r > 0$ gegeben.
    Wähle $n_0 ∈ ℕ$ mit
    \[
        \sum_{n=n_0}^\infty  2^{-n} < \frac r 2.
    \]

    mit $\epsilon  \coloneq \frac r 2 $ gilt dann
    \[
        \bigcap_{n=1}^{n_0} U(p_(,\epsilon ) ⊂ B_r(0).
    \]
    Sei dazu $x ∈ \bigcap_{n=1}^{n_0} U(p_n,\epsilon )$ beliebig.
    Dann ist
    \[
        d(x,0) \le \sum_{n=1}^{n_0} 2^{-n} \frac{p_n(x)}{1+p_n(x)} + \sum_{n=n_0}^\infty  2^{-n} < \epsilon  \sum_{n=1}^{n_0} 2^{-n} + \frac r 2 < \epsilon  + \frac r 2 = r,
    \]
    somit also $x ∈ B_r(0)$.
\end{proof}

\begin{bemerkung}
    Die Mengen $U(p_n,\epsilon _n)$ und deren endlichen Schnitte sind konvexe Mengen, das heißt
    \[
        x, y ∈ U(p_n,\epsilon _n),\alpha[0,1] \implies \alpha x+(1-\alpha )y ∈ U(p_n,\epsilon _n)
    \]
\end{bemerkung}
\begin{proof}
    Es ist
    \[
        p_n(\alpha x + (1-\alpha )y) \le |\alpha | \underbrace{p_n(x)}_{< \epsilon _n} + |1-\alpha |\underbrace{p_n(y)}_{< \epsilon _n} = \epsilon _n.
    \]
\end{proof}

Also besitzt der in \cref{satz-abzaehlbares-prod-seminormierter-raeume} gewonne metrische lineare Raum $(X,d)$ eine Umgebungsbasis von $0$, die nur aus konvexen elementen besteht.

\begin{definition}
    Ein topologischer linearer Raum $(X,\T)$, in dem jedes $x ∈ X$ eine Umgebungsbasis besitzt, die nur aus konvexen Mengen besteht, heißt \emph{lokalkonvex}.
\end{definition}

\begin{satz}
    Sei $X$ ein linearer Raum mit Semi-Normen $p_i, i ∈ I$, wobei $I$ eine beliebige Indexmenge ist, mit der Eigenschaft
    \[
        p_i(x) = 0 \text { für alle } i ∈ I \implies x = 0.
    \]
    Dann sind die Mengen
    \[
        U(p_i,\epsilon _i) = \{ x ∈ X: p_{(x) < \epsilon _i}\}, \quad \epsilon _i > 0, i ∈ I
    \]
    und deren endliche Schnitte eine konvexe Umgebungsbasis von $0 ∈ X$.
    Die dadurch gewonne Topologie $\T$ macht $X$ zu einem \emph{lokalkonvexen Hausdorff"=Raum}.
\end{satz}

\section{Beispiele}
Wir werden die unten angegebenen Beispiele auch gleich auf Vollständigkeit untersuchen.

\begin{definition}
    \begin{enumerate}
    \item
        Ein metrischer linearer Raum $(X,d)$ der vollständig ist, heißt \emph{Fréchet"=Raum}.
    \item
        Ein normierter Raum $(X,\norm\cdot)$, der vollständig ist, heißt \emph{Banach"=Raum}.

    \end{enumerate}
\end{definition}

\begin{beispiel-nn}[$\ell^p$-Räume]
    \begin{enumerate}
    \item 
        $(\ell^p,\norm\cdot_p)$, $1 \le p < \infty $ ist normierter Raum mit
        \[
            \norm x _p = \left( \sum_{i=1}^\infty  |x_i|^p \right)^{1/p}.
        \]
    \item 
        $(\ell^\infty ,\norm\cdot_\infty)$, ist normierter Raum mit $\norm x _\infty  = \sup_{i ∈ ℕ} |x_i|$.
    \item 
        $(\ell^p,|\cdot|_p = \norm\cdot_p^p)$, $0 \le p < 1$ ist quasi-normierter Raum.
    \end{enumerate}
\end{beispiel-nn}

\begin{bemerkung}
    Für $0 < p < q \le \infty $ gilt $\ell^p ⊂ \ell^q ⊂ \ell^\infty $.
\end{bemerkung}
\begin{beweis}
    Sei $x ∈ \ell^p$ mit $|x| = 1 = \sum_{i ∈ ℕ} |x_i|^p$.
    Dann ist für alle $i ∈ ℕ$ $|x_i|^p \le 1$, also auch $|x_i| < 1$.
    Dann folgt auch $\sum_{i ∈ ℕ} |x_i|^q < 1$, also $x ∈ \ell^q$ und $\sup_{i ∈ ℕ} |x_i| \le 1$, also $x ∈ \ell^\infty $.
\end{beweis}


\begin{satz}
    Für $1 \le p \le \infty $ ist $(\ell^p,\norm\cdot_p)$ ein Banachraum.
    Für $0 < p < \infty $ ist $(\ell^p,|\cdot|_p)$ ein Fréchet-Raum.
\end{satz}
\begin{proof}
    Nur für $1 \le p < \infty $.
    Sei dazu $(x_n)_{n ∈ ℕ}\ell^p$  eine Cauchy-Folge, also
    $x_n = (ξ_k^n)_{k ∈ ℕ}$ und für jedes $\epsilon  > 0$ gibt es ein $n_0$ mit
    \[
        ∀n,m > n_0: \norm{x_n-x_m}_p = \left( \sum_{k=1}^\infty  |ξ_k^n-ξ_k^m|^p \right)^{1/p} < \epsilon .
    \]
    Sei $k_0 ∈ ℕ$ beliebig. Dann ist $(ξ_k^n)_{n ∈ ℕ}$
    eine Cauchy-Folge in $\K$, besitzt also einen Grenzwert $ξ_{k_0}$.
    Setze nun $x \coloneq (ξ_k)_{k ∈ ℕ}\K^\infty  = s$. Wir vermuten $x$ als Grenzwert unserer Cauchy-Folge.
    Also müssen wir zeigen, dass $x ∈ \ell^p$, und dass unsere Folge tatsächlich gegen $x$ konvergiert.

    Es gilt
    \[
        \norm{x_n}_! \le \underbrace{\norm{x_n-x_{n_0}}}_{< \epsilon } + \norm{x_{n_0}} \quad \forall n \ge n_0
    \]
    Deshalb existiert ein $M > 0$ mit $\norm{x_n}_p < M$ für alle $n ∈ ℕ$, also
    \[
        \sum_{k=1}^N |ξ_k^n|p < \sum_{k =1}^\infty  |ξ_k^n|^p \le M^p < \infty .
    \]
    Also haben wir
    \[
        \sum_{k=1}^N |ξ_k^p| \le M^p \quad ∀ n ∈ ℕ,
    \]
    also durch Grenzwertbildung $N → \infty $ auch $\norm{x}_p^p \le M^p$ bzw. $x ∈ \ell^p$.


    Ferner haben wir
    \[
        \sum_{k=1}^N |ξ_k^n-ξ_k^m|^p < \epsilon ^p \quad ∀ N ∈ ℕ, n, m \ge n_0(\epsilon ).
    \]
    Für $n → \infty $ folgt
    \[
        \sum_{k=1}^N |ξ_k-ξ_k^m|^p < \epsilon ^p \quad ∀N ∈ ℕ, m \ge n_0, 
    \]
    und mit $N → \infty $
    \[
        \sum_{k=1}^\infty  |ξ_k-ξ_k^m|^p < \epsilon ^p \quad ∀m \ge n_0, 
    \]
    also die Konvergenz.
\end{proof}
\begin{beispiel-nn}
    Betrachte den Folgenraum $S = \K^\infty  = \{x = (ξ_n)_{n ∈ ℕ}, ξ_n ∈ \K\}$.
    Dann ist 
    \[
        p_n(x) \coloneq |ξ_n|, \quad p_n: \K^\infty  → ℝ
    \]
    eine abzählbare Familie von Halbnormen mit
    \[
        p_n(x) = 0 ∀n ∈ ℕ \implies x = 0 ∈ \K^\infty 
    \]
    Nach \cref{satz-abzaehlbares-prod-seminormierter-raeume} folgt, dass $(\K^\infty , d)$ mit
    \[
        d(x,y) \coloneq \sum_{n ∈ ℕ} 2^{-n} \frac{p_n(x-y)}{1+p_n(x-y)}
    \]
    ein metrischer linearer Raum ist.
    Der Konvergenzbegriff entspricht gerade der komponentenweisen Konvergenz, das heißt, für eine Folge $(x_k)_{k ∈ ℕ}$ mit $x_k = (ξ^k_n)_{n ∈ ℕ}$ gilt
    \begin{align*}
        x_k \xrightarrow[k→\infty ]{} 0
        &\gdw d(x_n,0) \xrightarrow[k→\infty ]{} 0 \\
        &\gdw p_n(x_k) \xrightarrow[k→\infty ]{} ∀ n ∈ ℕ \\
        &\gdw |ξ_n^k| \xrightarrow[k→\infty ]{} 0 ∀ n ∈ ℕ.
    \end{align*}

    Wir fragen uns nun, ob auf dem $\K^\infty $  auch eine Topologie existiert, so dass der induzierte Konvergenzbegriff der der gleichmäßigen Konvergenz in allen Komponenten entspricht.
    Also
    \[
        x_k \xrightarrow[k → \infty ]{\text{glm}} 0\K^\infty  \gdw\epsilon  > 0 ∃ k_0 ∈ ℕ: |ξ_n^k| < \epsilon  ∀ k \ge k_0 ∀n ∈ ℕ.
    \]
    Wenn $\K^\infty $ ein topologischer linearer Raum sein soll, ist das nicht möglich. Notwendig wäre, dass für eine Folge $x  ∈ \K^\infty $
    \[
        \alpha _k \xrightarrow[k → \infty ]{} 0 \text{ in } \K \implies \alpha _k x \xrightarrow[k→\infty ]{} \text{ in } X = \K^\infty .
    \]
    Wähle dazu die Nullfolge $(\alpha _k)_{k ∈ ℕ} = (1/k)_{k ∈ ℕ} ⊂ ℝ$, $x= (n)_{n ∈ ℕ} ⊂ X$. Dann ist
    \[
        \alpha _k x = (n/k)_{n ∈ ℕ}\K^\infty 
    \]
    zwar eine Nullfolge in $\K^\infty$ ist, diese Konvergenz ist jedoch nicht gleichmäßig in $n$.
    Man kann zeigen, dass $\K^\infty $ mit $d$ vollständig, also ein Fréchet-Raum, ist.
    Ist $\K^\infty $ auch normierbar?
    Also gibt es auf $\K^\infty $ eine Norm, welche die gleiche Topologie erzeugt wie die $d$?
    Auch  das ist nicht möglich:
\end{beispiel-nn}
\begin{lemma}
    \label{lemma-s-metrikkugeln-enthalten-unterraeme}
    In $(\K^\infty ,d)$ gilt:
    \begin{enumerate}
    \item
        $B_1(0) = \K^\infty $
    \item
        Betrachte den linearen Unterraum $M_{n_0} \coloneq \{ x = (ξ_n)_{n ∈ ℕ}$ mit $ξ_n = 0$ für $n = 1,…,n_0 \}$.
        Dann gibt es für jeden Radius $r > 0$  ein $n_0 ∈ ℕ$, so dass $M_{n_0} ⊂ B_{r}(0)$.
        Das heißt, jede noch so kleine Metrikkugel enthält einen nichttrivialen Unterraum.
    \end{enumerate}
\end{lemma}
\begin{proof}
    \begin{enumerate}
    \item
        Das ist trivial.
    \item
        Sei $r > 0$ gegeben.
        Wähle nun $n_0$, so dass $\sum_{n=n_0+1}^\infty  2^{-n} < r$.
        Dann gilt
        \[
            ∀ x ∈ M_{n_0}: d(x,0) =
            \sum_{n ∈ ℕ} 2^{-n} \frac{p_n(x)}{1+p_n(x)} =
            \sum_{n=n_0}^\infty  2^{-n} \frac{p_n(x)}{1+p_n(x)} \le
            \sum_{n=n_0}^\infty  2^{-n} < r.
        \]
    \end{enumerate}
\end{proof}

Wäre nun die Topologie auf $(\K^\infty ,d)$ nun auch von einer Norm erzeugt, dann wären die Normkugeln
\[
    B_r^{\norm\cdot}(0) = \{ x ∈ \K^\infty : \norm x < \tilde r \}
\]
auch eine Umgebungsbasis der Null.
Das heißt insbesondere würden wir zu jedem $\tilde r$ ein $r$ finden, so dass $0 ∈ B_r^d(0) ⊂ B_r^{\norm\cdot} (0)$.
Mit \cref{lemma-s-metrikkugeln-enthalten-unterraeme} folgt also
\[
    M_{n_0} ⊂ B_r(0) ⊂ B_r^{\norm\cdot}(0)
\]
für ein geeignetes $n_0$.
Sei nun ein $0  \ne x ∈ M_{n_0}$. Dann ist, da $M_{n_0}$ ein Unterraum ist, auch $\alpha x ∈ M_{n_0}$ für alle $\alpha\K$.
Das heißt,
\[
    |\alpha | \cdot \norm x = \norm{\alpha x} < \tilde r \text{ für alle } \alpha\K,
\]
was bereits $\alpha  = 0$ impliziert. Das ist ein Widerspruch.


\begin{beispiel-nn}[Räume beschränkter Funktionen]
    Sei $S$  eine beliebige Menge und $B(S) \coloneq \{ f: S → \K, f(s)$ ist beschränkt $\}$.
    Dann wird $B(S)$ mit
    \[
        \norm f _{B(S)} \coloneq \sup_{x ∈ S} |f(x)| < \infty ,
    \]
    der $\sup$-Norm, zu einem Banachraum.
    Dabei ist offensichtlich, dass $\norm\cdot_{B(S)}$ tatsächlich eine Norm ist, und wir werden in einer Übung zeigen, dass die induzierte Metrik tatsächlich vollständig ist.
\end{beispiel-nn}

\begin{lemma-nn}
    \label{lemma-vollst-ubertragt-abgeschl-teilmengen}
    Sei $(X,d)$ ein metrischer Raum, $Y ⊂ X$. Es gilt
    \begin{enumerate}
    \item
        Wenn $(X,d)$ vollständig ist und $Y$ abgeschlossen, dann ist auch $(Y,d|_{Y×Y}$ vollständig.
    \item
        Wenn $(Y,d|_{Y×Y}$ vollständig ist, so ist $Y$ abgeschlossen in $(X,d)$.
    \end{enumerate}
\end{lemma-nn}
\begin{proof}
    Übungsaufgabe.
\end{proof}


\begin{beispiel-nn}[Räume stetiger Funktionen]
    Sei $K ⊂ ℝ^n$ kompakt, also nach Heine-Borel abgeschlossen und beschränkt.
    Dann ist
    \[
        C(k) = \{ f: K → \K, f \text{ stetig} \}
    \]
    ein normierter Raum mit
    \[
        \norm{f}_{C(K)} = \norm{f}_{\infty } = \max_{t ∈ K} |f(t)|,
    \]
    der Maximumsnorm.
    Dieses Maximum wird tatsächlich immer angenommen, da $K$ kompakt ist (Satz von Minimum und Maximum).
    Insbesondere sind alle stetigen Funktionen auf  $K$ beschränkt. Damit gilt offensichtlich $C(K) ⊂ B(K)$ und $\norm{f}_{C(K)} = \norm{f}_{B(K)}$ für alle $f ∈ C(K)$.
    Da jede stetige Funktion auf kompakten Teilmengen von metrischen Räumen auch gleichmäßig stetig ist, das heißt
    \[\epsilon  > 0\delta  > 0: \left( |t_1-t_2| < \delta  \implies |f(t_1)-f(t_2)| < \epsilon  \right) ∀ t_1,t_2 ∈ K
    \]
\end{beispiel-nn}

\begin{lemma}
    $C(K)$ ist ein abgeschlossener Unterraum von $(B(K), \norm\cdot_{B(K)})$ und somit insbesondere auch (mit \cref{lemma-vollst-ubertragt-abgeschl-teilmengen}) vollständig.
\end{lemma}
\begin{proof}
    Sei $(f_i)_{i ∈ ℕ}$ eine konvergente (in $(B(K),\norm\cdot_{B(K)})$) Folge in $C(K)$.
    Dann existiert ein $f ∈ B(K)$ mit $f_i \xrightarrow[i → \infty ]{\norm{\cdot}_{B(K)}} f$.
    Wir müssen zeigen, dass $f$ bereits stetig ist.
    Für beliebige $t₁, t_2 ∈ K$ gilt
    \[
        |f(t_1)-f(t_2) | \le \underbrace{|f_i(t_1)-f_i(t_2)|}_{< \epsilon /3 \text{ für } |t_1-t_2| < \delta ^{(i)}(\epsilon )} + 2 \underbrace{\norm{f_i - f}_{B(K)}}_{< \epsilon /3 \text{ für } i > i_0}  < \epsilon .
    \]
    Damit ist $f$ auch gleichmäßig stetig, also insbesondere auch stetig und in $C(K)$.
\end{proof}
Das heißt, die Stetigkeit der Folgenglieder $(f_i)_{i ∈ ℕ} ⊂ C(K)$ überträgt sich auf die Grenzfunktion und Konvergenz in $(C(K),\norm\cdot_{\infty })$ ist „gleichmäßig auf $K$“.
Wegen  dieser Eigenschaft ist die Maximumsnorm $\norm\cdot_\infty $ auch die natürliche Norm auf $C(K)$.
Andere mögliche Normen (und damit andere Topologien) auf $C(K)$ wären z.B.
\[
    \norm{f}_p = \left(  \int_K |f(t)|^p dt \right)^{1/p}, \quad 1 \le p < \infty .
\]
Allerdings ist die Konvergenz in dieser Topologie impliziert keine Stetigkeit für die Grenzfunktion.


\begin{beispiel-nn}
    Sei $\Omega ⊂ ℝ^n$ offen und analog
    \[
        C(\Omega) \coloneq \{ f: \Omega\K, f \text { stetig }\}.
    \]
    Hier können Funktionen aber auch unbeschränkt sein. Also braucht $\sup |f|$ nicht mehr zu existieren.
\end{beispiel-nn}

\begin{definition}
    Es sei $(K_m)_{m ∈ ℕ}$ eine \emph{Ausschöpfung} von $\Omega$ mit kompakten Mengen $K_= ⊂ \Omega$, das heißt, es gelte
    \[
        \begin{cases}
            \Omega = \bigcup_{m ∈ ℕ} K_m, \quad K_m ⊂ K_{m+1}, \\
            K ⊂ \Omega \text { kompakt } \implies K ⊂ K_m \text { f ür ein } m ∈ ℕ
        \end{cases}
    \]
\end{definition}
Man nehme z.B.
\[
    K_m = \{ x ∈ \Omega ⊂ ℝ^n: \norm{x} \le m, \operatorname{dist}(x,∂\Omega) \ge 1/m\},
\]
wobei $\operatorname{dist}(x,∂\Omega) \coloneq \inf\{ \norm{x-y}: y ∈ ∂\Omega\}$ und $∂M = \cl \Omega \setminus \Omega$.

Dann ist $C(\Omega)$ mit der Metrik
\[
    d(f,0) = \sum_{m ∈ ℕ} 2^{-m} \frac{\norm{f}_{C(K_m)}}{1+\norm{f}_{C(K_m)}}
\]
ein Fréchetraum, also ein metrisierbarer linearer Raum nach \cref{satz-abzaehlbares-prod-seminormierter-raeume}, da
\[
    \norm{f}_{C(K_m)} = 0 ∀ m ∈ ℕ \implies f = 0 ∈ C(\Omega).
\]

Es gilt in diesem Raum
\[
    d(f_i,f) \xrightarrow[i → \infty ]{} 0 \gdw
    \norm{f_i-f}_{C(K_m)} \xrightarrow[i → \infty ]{} ∀m  ∈ ℕ,
\]
was ja gerade gleichmäßige Konvergenz auf jeder Kompakten Menge $K ⊂ \Omega$ bedeutet.
Damit ist Stetigkeit der Folgenglieder $(f_i)_{i ∈ ℕ} ⊂ C(\Omega)$ impliziert Stetigkeit der Grenzfunktion $f ∈ C(\Omega)$, da Stetigkeit nur eine lokale Eigenschaft ist.

Wir werden in der Übung sehen, dass $C(\Omega)$ mit dieser Metrik $d_{C(\Omega)}$ nicht normierbar ist.

\begin{beispiel-nn}[Räume differenzierbarer Funktionen]
    \begin{enumerate}
    \item
        Betrachte die Menge $C^\ell(K) = \{ f: K → ℝ, D^\alpha  f$ existiert und ist stetig für$|\alpha | < \ell \}$ der $\ell$-mal stetig differenzierbaren Funtktionen auf einer kompakten Menge $K ⊂ ℝ^n$ mit $\ell ∈ ℕ_0$
        Dabei ist $\alpha  = (\alpha _1,…,\alpha _n) ∈ ℕ_0^n$ ein Multiindex, $|\alpha | = \sum_{i=1}^n \alpha _i$ und
        \[
            D^\alpha  f = \frac{∂^{|\alpha |} f}{∂x_1^{\alpha _1}\cdots∂x_n^{\alpha _n}}.
        \]
        Dann wird $C^\ell(K)$ mit der Norm
        \[
            \norm{f}_{C^\ell(K)} = \max_{|\alpha | \le l} \max_{x ∈ K} | D^\alpha  f(x)|
        \]
        zu einem Banachraum. Die meisten Eigenschaften sind klar, die Vollständigkeit folgt unmittelbar aus der Vollständigkeit von $C(K)$
        Konvergenz in $C^\ell(K)$ bedeutet gerade gleichmäßige Konvergenz aller partiellen Ableitungen bis zur Ordnung $\ell$ auf ganz $K$.
    \item
        Sei $\Omega ⊂ ℝ^n$ offen und
        $\C^\ell(\Omega) = \{ f: \Omega → ℝ, D^\alpha  f$ existiert und ist stetig für$|\alpha | < \ell \}$
        der Raum der $\ell$-mal stetig differenzierbaren Funtktionen auf $\Omega$ mit $\ell ∈ ℕ_0$.
        $C^\ell(\Omega)$ wird mit der Metrik
        \[
            d(f,g) \coloneq \sum_{m ∈ ℕ} 2^{-m} \frac{p_{m,l}(f-g)}{1+p_{m,l}(f-g)}, \quad p_{m,l}(f) = \max_{|\alpha | \le \ell} \norm{D^\alpha  f}_{C(K_m)},
        \]
        wobei die $K_m$ Ausschöpfungen von $\Omega$ mit kompakten Mengen sind, zu einem Fréchetraum.
        Konvergenz in $C^\ell(\Omega)$ bedeutet gerade gleichmäßige Konvergenz aller partiellen Ableitungen bis zur Ordnung $\ell$ auf jedem Kompaktum, das in $\Omega$ enthalten ist.
        Auch dieser Raum ist nicht normierbar mit einem analogem Argument wie bei den stetigen Funktionen auf $\Omega$.
    \item
        Wir betrachten nun einige Unterräume von $\C^\ell(\Omega)$:
        \begin{enumerate}[label=(\roman*)]
        \item
        $\C^\ell_B(\Omega) = \{ f: \Omega → ℝ, D^\alpha  f$ existiert, ist beschränkt und ist stetig für$|\alpha | < \ell \}$
        wird zum normierten Raum mit 
        \[
            \norm{f}_{C^\ell_B(\Omega)} = \max_{|\alpha | \le l} \sup_{x ∈ \Omega} | D^\alpha  f(x)|
        \]
        Zwar gilt $C^\ell_B(\Omega) ⊂ C^\ell(\Omega)$ (als Mengen), jedoch besitzt $C^\ell_B(\Omega)$ nicht die Relativtopologie von $\C^\ell(\Omega)$, wie wir in einer Übung sehen werden.
        \begin{definition}
            \begin{enumerate}
            \item 
                Für $\Omega ⊂ ℝ^n$ offen und $f: \Omega → ℝ$ heißt
                \[
                    \supp f \coloneq \cl{ \{ x ∈ \Omega, f(x) \ne 0 \}}
                \]
                der \emph{Träger} oder \emph{Support} von $f$.
            \item
                Wir sagen für eine Menge $M ⊂ \Omega$ \emph{$M$ liegt kompakt in $\Omega$}, wenn $\cl M $ kompakt ist und $\cl M ⊂ \Omega$. Wir schreiben dafür $M ⊂⊂ \Omega$.
            \end{enumerate}
        \end{definition}
    \item
        $C_0^\ell(\Omega) = \{ f ∈ C^\ell(\Omega) : \supp f ⊂⊂ M \}$
        Funktionen mit $\supp f ⊂⊂ M $ haben Luft zum Rand von $\Omega$:
        \[
            \operatorname{dist}(\supp(f), ∂\Omega) > 0,
        \]
        denn sowohl $\supp f$ als auch $∂\Omega$ sind abgeschlossen.
        Es gibt verschiedene Möglichkeiten, Topologien für $C_0^\ell(\Omega)$ zu wählen:
        \begin{enumerate}
        \item
            $C_0^\ell(\Omega) ⊂ C^\ell(M)$ mit Spurtopologie von der Metrik.
        \item
            $C_0^\ell(\Omega) ⊂ C_B^\ell(M)$ mit Spurtopologie von der Norm.
        \end{enumerate}
        Diese Topologien sind jedoch nicht identisch.
        \end{enumerate}
    \item
        Sei $\Omega ⊂ ℝ^n$ offen und 
        $C^\infty (\Omega) = \{ f: \Omega → ℝ, D^\alpha f $ existiert und ist stetig für alle $\alpha  ∈ ℕ_0^n \} = \bigcap_{\ell ∈ ℕ}C^\ell(\Omega)$.
        Wir definieren die Topologie wieder über eine Metrik durch Seminormen
        \[
            d(f,g) \coloneq \sum_{m ∈ ℕ} 2^{-m} \frac{p_{m}(f-g)}{1+p_{m}(f-g)}, \quad p_{m}(f) = \max_{|\alpha | \le m} \norm{D^\alpha  f}_{C(K_m)}.
        \]
        Mit dieser Metrik wird $C^\ell(\Omega)$ zum Fréchetraum.
        Konvergenz in $C^\infty (\Omega)$ bedeutet gerade gleichmäßige Konvergenz aller partiellen Ableitungen auf jedem Kompaktum, das in $\Omega$ enthalten ist.
        Auch dieser Raum ist nicht normierbar mit einem analogem Argument wie bei den stetigen Funktionen auf $\Omega$.
    \item
        Sei $\Omega ⊂ ℝ^n$ offen und $C_0^\infty (\Omega) = \{ f ∈ C^\infty (\Omega) : \supp f ⊂⊂ M \}$ der \emph{Raum der Testfunktionen}.
        Ein Beispiel für so eine Funktion ist
        \[
            f(x) =
            \begin{cases}
                c \exp \left( - \frac{1}{{1-|x|^2}} \right), & |x| < 1 \\
                0, & |x| \ge 1
            \end{cases},
        \]
        wobei $\Omega = B^{\norm\cdot_\infty}_2(0)$, $|\cdot| = \norm\cdot_2$ und $c ∈ ℝ$ konstant.
        Offensichtlich ist $C_0^\infty (\Omega) ⊂ C^\infty (\Omega)$.
        Wenn man auf $C_0^\infty (\Omega)$ jedoch die Spurtopologie wählt, bekommt man später Probleme (bestimmte Funktionale auf $C_0^\infty (\Omega)$ sind nicht mehr stetig, wie wir in einer Übungsaufgabe sehen werden.
        Man nennt Funktionale auf $C_0^\infty (\Omega)$ auch Distributionen).
        Außerdem wäre der $C_0^\infty (\Omega)$ mit dieser Metrik nicht vollständig -- der Träger der Grenzfunktion muss nicht mehr beschränkt sein.
        \begin{definition-nn}
            Sei $M ⊂ X$ und $X$ ein linearer Raum. Dann heißt
            \[
                \conv (M) \coloneq \{ x: ∃\alpha _i > 0, x_i ∈ M, i ∈ \{1,…,k\}: \sum_{i=1}^k \alpha _i = 1, \sum_{i=1}^k \alpha _i x_i = x \}
            \]
            die \emph{konvexe Hülle} von $M$.
        \end{definition-nn}
        Aus Gründen, die erst später zu verstehen sind, wählt man auf $C^\infty _0(\Omega)$ folgende lokalkonvxe Topologie:
        Setze
        \[
            p(\xi) \coloneq \sum_{k ∈ ℕ} 2^{-k} \frac{\norm \xi _{C^k(\Omega)}}{1 + \norm \xi _{C^k(\Omega)}}, \quad \xi ∈ C_0^\infty (\Omega)
        \]
        Sei $(D_j)_{j ∈ ℕ}$ eine Ausschöpfung von $\Omega$ mit offenen Mengen, also $D_j ⊂ D_{j+1}, D_j ⊂⊂ \Omega, \bigcup_{j ∈ ℕ} D_j = \Omega$.
        Eine mögliche Wahl wäre beispielsweise $D_j = K_j^\circ$, wobei die $K_j$ wie oben sind.
        Für $\epsilon  = (\epsilon _j)_{j ∈ ℕ} ∈ ℝ^\infty , \epsilon _j > 0$ für alle $ℕ$ definieren wir eine Umgebungsbasis der $0 ∈ C_0^\infty (\Omega)$  durch alle Mengen
        \[
            U_\epsilon  \coloneq \conv \left[ \bigcup_{j ∈ ℕ}  \{ \xi ∈ C^\infty _0 : p(\xi) < \epsilon _j \} \right] ⊂ C_0^\infty (\Omega).
        \]
        mit $\epsilon  = (\epsilon _j)_{j ∈ ℕ} ∈ ℝ^\infty , \epsilon _j > 0$ und endliche Schnitte davon. Andere Umgebungen umgeben sich durch Translation.
        Die so definierte Topologie nennen wir $\T_\D$ und den Raum $C_1^\infty (\Omega)$  auch $\D(\Omega)$.
        Es stellt sich heraus, dass diese Topologie tatsächlich unabhängig von der gewählten Ausschöpfung ist.
        Außerdem ist $(\D(\Omega),\T_\D)$ ein topologischer linearer Raum, das heißt, die Vektorraumoperationen sind stetig.
        \begin{lemma}[Charakterisierung offener Mengen in $\D(\Omega)$]
            Es gilt
            \[
                O ∈ \T_\D \iff ∀ ξ ∈ O ∃ \epsilon =(e_j)_{j ∈ ℕ} ∈ ℝ^\infty , e_j > 0: e+U_\epsilon  ⊂ O.
            \]
            Das heißt, die Topologie $\T_\D$ und die Topologie
            \[
                \tilde T_\D = \{ O ⊂ C_0^\infty (\Omega): ∀ ξ ∈ O ∃ \epsilon  = (\epsilon _j)_{j ∈ ℕ} ∈ ℝ^\infty , \epsilon _j > 0: \epsilon + U_\epsilon  ⊂ O \}
            \]
            sind gleich.
        \end{lemma}
        \begin{proof}
            Übung.
        \end{proof}
        \begin{korollar}
            Die Mengen $U_\epsilon$ sind bereits eine Umgebungsbasis der Null.
            Nach Definition sind sie aber auch Konvex, das heißt $(\D(\Omega),\T_\D)$ ist ein lokalkonvexer Hausdorff-Raum.
        \end{korollar}
        \begin{satz}
            $ξ_m \xrightarrow[m → \infty ]{} 0 \gdw$
            \[
                \begin{cases}
                    (i), & \text{Es existiert $D$ offen mit $D ⊂⊂ \Omega$ und
                        $ξ_m ∈ C_0^\infty (D)$ für alle $m ∈ ℕ$} \\
                    (ii), & \text{Für jedes $k ∈ ℕ$ gilt:
                        $\norm{ξ_m}_{C^k(\cl{\Omega})} \xrightarrow[m → \infty ]{} 0$}
                \end{cases}
            \]
        \end{satz}
        \begin{proof}
            Zeige nur „$\Leftarrow$“. Sei dazu $(ξ_m)_{m ∈ ℕ}$ eine Folge mit (i) und (ii). 
            Wähle nun $D_j$ von oben mit $D ⊂ D_j$ ($j$ ist fest).
            Sei nun $\epsilon =(\epsilon _i)_{i ∈ ℕ}, \epsilon _i > 0$ gegeben. Dann müssen wir zeigen, dass für alle $m > m_0$ schon $ξ_m ∈ U_\epsilon $  gilt.
            Zunächst sind nach (i) $ξ_m ∈ C^\infty _0(D_j)$ .
            Außerdem gilt
            \[
                p(\xi_m) \le \underbrace{\sum_{k=1}^N 2^{-k} \frac{ \norm{\xi_m}_{C^k(\cl \Omega)} } {1+ \norm{\xi_m}_{C^k(\cl \Omega)} }}_{\text{wegen (i)} < \epsilon _j/2 \text{ für $m \ge m_0(\epsilon _j,N)$}}  + \underbrace{\sum_{k=N+1} 2^{-k}}_{<\epsilon _j/2 \text{ für $n$ groß genug}} < \epsilon _j.
            \]
        \end{proof}
    \item
        Betrachten wir nun Lebesgue-integrierbare Funktionen.
        Bereits eingeführt wurden die Räume $\L^p(\Omega)$ und $L^p(\Omega)$, $0 < p < \infty $, wobei $\Omega ⊂ ℝ^n$ offen.
        Diese sind für $1 \le p < \infty $ normiert, und für $0 < p < 1$ quasi-normiert.
        Für $p = \infty $ setzen wir
        \[
            \L^\infty (\Omega)  \coloneq \{ f: \Omega → ℝ ∪ \{ -\infty , \infty  \}, f \text{ messbar und fast überall beschränkt} \}.
        \]
        Damit haben wir offenbar
        \[
            C(\Omega) ∩ B(\Omega) ⊂ \L^\infty (\omega).
        \]
        Sei
        \[
            \norm f _{\L^\infty (\Omega)} \coloneq \supess_{t ∈ \Omega} |f(t)| \coloneq \inf_{M ⊂ \Omega \text{ NM}} \sup_{t ∈ \Omega \setminus M} |f(t)|.
        \]
        Dann gilt für $f ∈ \L^\infty (\Omega)$
        \[
            \norm f = 0 \gdw f = 0 \text{ fast überall}
        \]
        Mit $N \coloneq \{ f ∈ \L^\infty (\Omega) : \norm f = 0 \}$ wird
        \[
            L^\infty (\Omega) \coloneq \left( \L^\infty (\Omega)/N, \norm\cdot_{L^\infty (\Omega)} \right)
        \]
        zu einem normiertem Raum.
    \end{enumerate}
\end{beispiel-nn}


{ \LARGE Vorlesung vom Donnerstag, 9. November fehlt (genauso wie vermutlich alle weiteren Donnerstagsvorlesungen ab jetzt)}

Seien $f_n → f ∈ L^p(\Omega), h ∈ C_0^\infty (\Omega)$.
Dann
\[
    \lim_{n → \infty } ∫_\Omega f_n(t) h(t) dt = ∫_\Omega f(t) h(t) dt,
\]
denn
\begin{align*}
  ∫_\Omega (f_n(t) - f(t)) h(t) dt &\le ∫_{\supp h} M |f_n(t) - f(t)| dt \\
                              & \stackrel{\mathclap{\text{Hölder}}}{\le} \; M [ \supp(h)]^{1/q} \norm{f_n-f}_{L^p(\Omega)}0.
\end{align*}

\section{Beschränkte und kompakte Mengen in metrischen linearen Räumen}

Wir wissen bereits nach dem Satz von Heine-Borel, dass eine Teilmenge $K ⊂ ℝ^n$  genau dann kompakt ist, wenn sie abgeschlossen und beschränkt ist.
Beschränktheit bedeutet hier Beschränktheit in einer (beliebigen, da alle äquivalent) Norm.

Nun wollen wir so ein Konzept für Beschränktheit auch in allgemeinen metrischen (topologischen) linearen Räumen finden.

\begin{problem-nn}
    Die natürliche Übertragung $d(x,0) \le M$, $x ∈ B$ definiert \emph{keine} Beschränktheit.
    Gründe dafür sind:
    \begin{enumerate}
    \item
        In einigen metrischen Räumen gilt ohnehin $d(x,0) \le 1$ für alle $x ∈ X$.
    \item
        Ist $d$ eine Metrik auf $X$. Dann ist $\tilde d \coloneq \frac d {1+d} \le 1$ eine zu $d$ äquivalente Metrik auf $X$, wie wir in Topologie gesehen haben.
    \end{enumerate}
\end{problem-nn}

\begin{definition}
    Sei $(X,\T)$ ein topologischer linearer Raum, $B ⊂ X$ heißt \emph{beschränkt}, falls zu jeder offenen Umgebung $U$ von $0 ∈ X$ ein $\alpha  > 0$ existiert, so dass $B ⊂ \alpha U = \{\alpha u: u ∈ U\}$, das heißt jede Nullumgebung lässt sich so weit „aufblasen“, dass sie $B$ überdeckt.
\end{definition}

\begin{bemerkung-nn}
    Der Begriff „Beschränktheit“ hängt also von der Topologie ab.
\end{bemerkung-nn}


\begin{satz}
    Sei $(X,d)$ ein metrischer linearer Raum, dessen Metrik gemäß \cref{satz-abzaehlbares-prod-seminormierter-raeume} von abzählbar vielen Seminormen $(p_n)_{n ∈ ℕ}$ induziert ist.
    Dann ist eine Teilmenge $B ⊂ X$ genau dann beschränkt, wenn für jedes $k ∈ ℕ$ ein $M_k > 0$ existiert mit $p_k(x) \le M_k$ für alle $x ∈ B$.
\end{satz}
\begin{proof}
    „⇒“: Sei $k ∈ ℕ$ gegeben.
    Setze $r_k \coloneq \frac 1 {2^{k+1}}$ und $U \coloneq B_{r_k}(0)$.
    Da $B$ beschränkt ist, gibt es $\alpha  = \alpha _k > 0$, dass
    \begin{align*}
    & B ⊂ \alpha U = \alpha  B_{r_k}(0) \\
      \gdw & \alpha ^{-1} B ⊂ B_{r_k} (0) \\
      \gdw &d(\alpha ^{-1} x, 0) < r_k ∀ x ∈ B
    \end{align*}
    Dann gilt schon $p_k(x) \le M_k \coloneq \alpha _k$ für alle $x ∈ B$, denn
    \[
        \frac 1 {2^{k+1}} = r_k > d(\alpha _k^{-1} x, 0
        \ge 2^k \frac {p_k(\alpha _k^{-1}x)}{1+p_k(\alpha _k^{-1} x)}
        = 2^{-k} \frac{\alpha _k^{-1} p_k(x)}{1+\alpha _k^{-1} p_k(x)}.
    \]
    Also mit $\eta \coloneq \alpha _k^{-1} p_k(x)$ gilt $\frac 1 2 > \frac \eta {1+\eta}$, also $\eta < 1$ oder $p_k(x) \le M_k$ für alle $x ∈ B$.

    „⇐“:
    Sei also $p_k(x) \le M_k$ für alle $x ∈ B$ und $k ∈ ℕ$.
    Wir müssen nun zeigen, dass es für jedes $r > 0$ ein $\alpha  > 0$ gibt mit $B ⊂ \alpha B_r(0)$, also $\alpha ^{-1} B ⊂ B_r(0)$.
    Sei also $r > 0$ gegeben.
    Wähle nun $m_0 ∈ ℕ$ mit $\sum_{n=m_0+1}^\infty  2^{-n} < r/2$.
    Wähle $\alpha  > 0$ mit $\sum_{n=1}^{m_0} 2^{-n} \frac{\alpha ^{-1} M_k}{1+\alpha ^{-1} M_k} < r/2$.
    Dann gilt für alle $x ∈ B$
    \begin{align*}
        d(\alpha ^{-1} x, 0) &=
        \sum_{n ∈ ℕ} 2^{-n} \frac{\alpha ^{-1} p_n(x)}{1+\alpha ^{-1} p_n(x)} \\
        &\le \sum_{n=1}^{m_0} 2^{-n} \frac{\alpha ^{-1} p_n(x)}{1+\alpha ^{-1} p_n(x)} + \sum_{n=m_0+1}^\infty  2^{-n}  < r/2 + r/2 = r.
    \end{align*}
\end{proof}

\begin{korollar}
    Sei $(X,\norm\cdot)$ ein normierter linearer Raum, 
    Dann ist eine Teilmenge $B ⊂ X$ genau dann beschränkt, wenn $M > 0$ existiert mit $\norm{x} \le M$ für alle $x ∈ B$.
\end{korollar}
\begin{proof}
    Wähle $p_1(x) = \norm x$ und $p_k \equiv 0$ für $k \ge 2$ und verwende den vorherigen Satz.
\end{proof}


\begin{bemerkung}
    Kugeln $B_r(0)$ in $(X,d)$, wobei $d$ wie in \cref{satz-abzaehlbares-prod-seminormierter-raeume}, sinid also immer unbeschränkt,
    weil nichttriviale Unterräume $M_{n_0} ⊂ B_r(x)$ existieren.
    Insbesondere ist dies gültig in den Räumen $\K^\infty , C(\Omega), C^\ell(\Omega)$ und $C^\infty (\Omega)$.
\end{bemerkung}

\begin{definition}
    Ein topologischer linearer Raum $(X,\T)$ heißt \emph{lokalbeschränkt}, falls $0 ∈ X$ eine beschränkte Umgebung besitzt.
\end{definition}

\begin{bemerkung}
    Normierte Räume sind lokalbeschränkt und lokalkonvex. Es gilt aber auch die Umkehrung:
\end{bemerkung}

\begin{satz}[Kolmogorov]
    Ein topologischer linearer Raum $(X, \T)$ ist genau dann normierbar, das heißt, die Topologie wird von einer Norm induziert,
    wenn $(X,\T)$ lokalkonvex und lokalbeschränkt ist.
\end{satz}

\begin{beispiel-nn}
    Die Räume $\K^\infty , C(\Omega), C^\ell(\Omega)$ und $C^\infty (\Omega)$ sind nicht lokalbeschränkt, aber lokalkonvex. Somit sind sie auch nicht normierbar.
    Auch $L^p(0,1)$ mit $0 < p < 1$ ist nicht lokalkonvex, aber lokalbeschränkt, also nicht normierbar.
\end{beispiel-nn}

\begin{definition}
    Sei $(X,\T)$ ein topologischer linearer Raum.
    Eine Umgebung $U$ der Null heißt \emph{kreisförmig} oder \emph{balanced}, falls
    \[
        t U ⊂ U, \quad |t| < 1
    \]
\end{definition}

\begin{lemma}
    Sei $(X,\T)$ ein topologischer linearer Raum.
    Dann besitzt die Null eine Umgebungsbasis aus kreisförmigen Mengen.
\end{lemma}
\begin{proof}
    Übung.
\end{proof}

\begin{warnung-nn}
    Metrikkugeln müssen im Allgemeinen nicht kreisförmig sein (obwohl die uns bekannten Kugeln dies sind).
    Gegenbeispiel: $X = ℝ$, $d(x,y) \coloneq \left| ∫_x^y 1+\ind{ℝ_-}(s)\; ds \right|$.
\end{warnung-nn}

\begin{lemma}
    Sei $(X,\T)$ ein topologischer linearer Raum und $V ∈ \T$ eine Umgebung der 0.
    Dann gilt
    \[
        X = \bigcup_{n ∈ ℕ} n V.
    \]
\end{lemma}
\begin{proof}
    „$\supset$“: klar.

    „⊂“: Sei $x ∈ X$. Setze $β_n = 1/n, n ∈ ℕ$. Dann gilt
    \[
        β_n x \xrightarrow[n → \infty ]{} 0,
    \]
    also $β_n ∈ V$ für $n \ge n_0$. Damit haben wir aber $x ∈ n_0 V$.
\end{proof}

\begin{satz}
    Sei $(X,\T)$ ein topologischer linearer $T_2$-Raum und $K ⊂ X$ kompakt.
    Dann ist $K$ abgeschlossen und beschränkt.
\end{satz}
\begin{definition-nn}
    Falls auch die Umkehrung gilt, dann besitzt $X$ die \emph{Heine"=Borel"=Eigenschaft}.
\end{definition-nn}
\begin{warnung-nn}
    Im Allgemeinen ist die Umkehrung falsch.
\end{warnung-nn}
\begin{proof}
    Nach einer Übungsaufgabe ist $K$ bereits abgeschlossen.
    Also müssen wir nur zeigen, dass $K$ auch beschränkt ist.
    Sei $V ∈ \T$ eine Nullumgebung.
    Sei $W ⊂ \T$ eine kreisförmige Umgebung der $0$, die ganz in $V$ enthalten ist.
    Da
    \[
        K ⊂ X = \bigcup_{n ∈ ℕ} n W
    \]
    eine offene Überdeckung von $K$ ist, besitzt diese wegen $K$ kompakt eine endliche Teilüberdeckung
    \[
        K ⊂ \bigcup_{i=1}^s n_i W \stackrel{(*)}= n_s W, \quad n_1 < n_2 < … < n_s,
    \]
    also folgt die Behauptung mit $\alpha  = n_s$. $(*)$ gilt wegen der Kreisförmigkeit und $\left|\frac {n_i}{n_s}\right| \le 1$.
\end{proof}
\begin{bemerkung-nn}
    Ohne die Hausdorff-Eigenschaft gilt dies nicht. Gegenbeispiel: $X = ℝ$ mit der Klumpentopologie.
    Dann ist jede Teilmenge von $ℝ$ kompakt, aber nur $\emptyset$ und $ℝ$ sind abgeschlossen.
\end{bemerkung-nn}

\begin{definition}
    \begin{enumerate}
    \item 
        In einem topologischen Raum $(X,\T)$ heißt eine Menge $A ⊂ X$ \emph{relativ kompakt}, falls $\cl A$ kompakt ist.
    \item
        In einem metrischen Raum $(X,d)$ heißt eine Menge $A ⊂ X$ \emph{präkompakt}, falls für jedes $\epsilon  > 0$ die Menge $A$ von endlich vielen Bällen mit Radius $\epsilon $ überdeckt werden kann.
    \end{enumerate}
\end{definition}

\begin{satz}
    Sei $(X,d)$ ein metrischer Raum, $\emptyset \ne A ⊂ X$. Dann sind äquivalent:
    \begin{enumerate}
    \item
        $A$ ist kompakt.
    \item
        $A$ ist folgenkompakt.
    \item
        $(A,d|_{A×A})$ ist vollständig und $A$ präkompakt.
    \end{enumerate}
\end{satz}
\begin{proof}
    $(a) \iff (b)$ wurde bereits gezeigt. Wir zeigen nur $(b) ⇒ (c)$:
    Sei $(x_n)_{n ∈ ℕ} ⊂ A$ eine Cauchy-Folge. Nach (b) besitzt $(x_n)_{n ∈ ℕ}$ einen Häufungspunkt $x^*$.
    Da $(x_n)_{n ∈ ℕ}$ Cauchy-Folge ist, konvergiert $(x_n)_{n ∈ ℕ}$ schon gegen $x^*$. Damit ist $A$ vollständig.

    Angenommen, $A$ wäre nicht präkompakt. Dann gibt es $\epsilon  > 0$, so dass $A$ keine endliche Überdeckung mit $\epsilon $-Kugeln besitzt.
    Dadurch kann man eine Folge $(x_k)_{k ∈ K}$ definieren, mit $d(x_k,x_j) > \epsilon $ für $k \ne j$.
    Dann besitzt $(x_k)_{k ∈ K}$ offensichtlich keine Cauchy-Teilfolge, also auch keinen Häufungspunkt.
    Also $A$ präkompakt.
\end{proof}

Hier fehlt eine Vorlesung.

\section{?}

\begin{satz}
    3.6.4.    
\end{satz}
Hier gilt $M = \inf \{ c \ge 0:$ mit $C$ gilt (5) $\}$.
\begin{proof}
    $(1) \iff (2)$ schon gezeigt.

    $(3) \iff (4)$ klar durch die Charakterisierung von beschränkten Mengen in
    normierten Räumen und Ausnutzung der Linearität.

    $(2) \Rightarrow (4)$. Sei $T$ stetig in $x^*$. Wähle $\epsilon  > 0$, so dass $T(\cl B_\epsilon (x^*)) ⊂ B_1(T(x^*))$.
    Dann gilt für alle $x ∈ \cl B _1 (0)$
    \[
        x^* + \epsilon  x ∈ \cl B_\epsilon (x^*)
    \]
    und $T(x^*) + \epsilon T(x) = T(x^* + \epsilon x) ∈ B_1(T(x^*))$, das heißt $\epsilon  T(x) ∈ B_1(0)$ oder $\norm{T(x)}_Y \le \frac 1 {\epsilon } =: M$

    $(4) \Rightarrow (5)$. Für $x \ne 0$ gilt
    \[
        \norm{T(x)} \le \norm x \norm{T\left( \frac x {\norm x} \right)} \le M \norm x,
    \]
    also gilt die Aussage mit $C \coloneq M$.

    $(5) \Rightarrow (1)$. Für $x, x_1 ∈ X$ gilt
    \[
        \norm{T(x) - T(x_1)} = \norm {T(x-x_1)} \le C \norm x-x_1 \xrightarrow[x → x_1]{} 0.
    \]
    Damit ist $T$ stetig in $x_1$.
\end{proof}

\begin{korollar}
    Sei die Situation wie in 6.4 Ist $T$ zusätzlich bijektiv, so ist $T$ genau dann ein Homöomorphismus, wenn es Konstanten $m, M > 0$ gibt mit
    \[
        m \norm x \le \norm {T(x)} \le M \norm {x}
    \]
    für alle $x ∈ X$
\end{korollar}
\begin{beweis}
    klar.
\end{beweis}

\begin{warnung-nn}
    $T$ linear, bijektiv und stetig impliziert selbst in normierten Räumen noch nicht, dass auch die Inverse Abbildung $T^{-1}$ auch stetig ist, wie wir in der Übung sehen werden.
    Sind $X$ und $Y$ aber Banachräume, so gilt dies aber (Satz von der offenen Abbildung).
\end{warnung-nn}

Nun zur Charakterisierung von Stetigkeit in metrischen linearen Räumen.

\begin{satz}
    Sei $T: X → Y$ linear, $X$, $Y$ lineare metrische Räume.
    Dann ist $T$ genau dann stetig, wenn $T$ beschränkt ist.
\end{satz}

In topologischen linearen Räumen gilt dies jedoch nciht.

\begin{satz}
    3.6.7
\end{satz}
\begin{proof}
    Nur „$\Leftarrow$“: Nach 6.6 reicht es, Beschränktheit von $T$ zu zeigen, also dass, wenn $B ⊂ X$ beschränkt ist, auch $T(B) ⊂ Y$ beschränkt ist.
    $B ⊂ X$ ist genau dann beschränkt, wenn für alle $k ∈ ℕ$ $C_k > 0$ existieren mit $p_k(x) \le C_k$ für alle $x ∈ B$.
    Nach Voraussetzung ist dann aber auch für alle $x ∈ B$
    \[
        q_m(Tx) \le M_m(C_{n_1} + … + C_{n_k}) =: K_m,
    \]
    was nach 5.2 heißt, dass $T(B)$ beschränkt in $Y$ ist.
\end{proof}

\begin{definition}
    Seien $X, Y$ topologische lineare Räume. Dann bezeichnet $\L(X, Y) \coloneq \{ T: X → Y: T$ linear und stetig $\}$ den \emph{Raum der stetigen (beschränkten) Operatoren}.
    Im Spezialfall $Y = \K$ sei $X' \coloneq \L(X, \K)$ der \emph{Raum der stetigen Funktionale} oder auch der \emph{Dualraum von $X$}.
\end{definition}
\begin{bemerkung-nn}
    \begin{enumerate}
    \item
        $\L(X,Y)$ ist wieder ein linearer Raum.
    \item
        Metrische lineare Räume haben Dualräume, die im Allgemeinen nicht mehr metrisierbar sind.
    \item
        $X' = \{ 0\}$ ist möglich, wie wir in der Übung sehen werden
    \item
        Ist  $X$ jedoch normierbar, so folgt aus den Hahn-Banach-Sätzen, dass $X'$ nichttrivial ist.
    \item
        Falls $X$ und $Y$ normierte Räume sind, dann wird $\L(X, Y)$ ebenfalls zu einem normierten Raum mit der Operatornorm
        \begin{align*}
            \norm T &\coloneq \norm T _{\L(X,Y)} \coloneq \sup \{\norm x _X \le 1\} \norm {Tx}_Y \\ &= \inf \{ C \ge 0: ∀x ∈ X: \norm {Tx} \le C \norm x \}.
        \end{align*}
        Das heißt, wir haben
        \begin{equation}
            \label{eq:61}
            ∀x ∈ X: \norm {Tx}_Y \le \norm T \norm x _X
        \end{equation}
        Also haben wir
        \[
            \norm{(T_1 + T_2)x} = \norm{T_1x + T_2x} \le \norm{T_1x} + \norm{T_2x} \le \left( \norm{T_1} + \norm{T_2} \right) \norm{x},
        \]
        und somit $T_1 + T_2 ∈ \L(X,Y)$ und $\norm{T_1 + T_2} \le \norm{T_1} + \norm{T_2}$ nach \eqref{eq:61}.
    \item
        Auf $\L(ℝ^n,ℝ^m)$ ergeben sich die bekannten Matrixnormen.
    \end{enumerate}
\end{bemerkung-nn}

\begin{satz}
    Seien $X, Y$ normierte Räume, $Y$ vollständig. Dann ist $\L(X,Y)$ ein Banachraum.
    Insbesondere ist $X'$ immer ein Banachraum.

    Sei $Z$ ebenfalls ein normierter Raum.
    Ist $T ∈ \L(X,Y)$, $S ∈ \L(Y,Z)$, so ist $ST ∈ \L(X,Z)$ und $\norm{ST}_{\L(X,Z)} \le \norm S \norm T$.
\end{satz}

\begin{proof}
    Es ist nur noch die Vollständigkeit zu zeigen.
    Sei dazu $(T_n)_{n ∈ ℕ}$ eine Cauchy-Folge in $\L(X,Y)$.
    Das heißt, für jedes $\epsilon  > 0$ existiert ein $N_0$ mit $\norm {T_n - T_m} < \epsilon $ für $n, m > N_0$.
    Also mit \eqref{eq:61} $\norm {T_n x - T_mx} \le \norm {T_n - T_m} \norm x < \epsilon  \norm x$ für alle $x ∈ X$ und $n,m > N_0$.
    Insbesondere ist $(T_nx)_{n ∈ ℕ}$ eine Cauchy-Folge in $Y$. Da $Y$ vollständig ist, besitzt diese Folge einen Grenzwert $y_x ∈ Y$.
    Wir definieren eine Abbildung
    \[
        T: X → Y, x ↦ y_x.
    \]
    Dann ist $T$ linear, weil alle $T_n$ linear sind. Also ist nur die Stetigkeit von $T$ und die Konvergenz von $(T_n)_{n ∈ ℕ}$ gegen $T$ zu zeigen.
    Für die Stetigkeit bekommt man unter Verwendung der Dreicksunglechung direkt
    \[
        \left| \norm {T_n} - \norm{T_m} \right| \le \norm {T_n - T_m} < \epsilon  \quad ∀ n, m \ge N_0,
    \]
    also eine Cauchyfolge $\left( \norm{T_n} \right)_{n ∈ ℕ}$ in $ℝ$, die wegen der Vollständigkeit von $ℝ$ konvergent, also insbesondere auch beschränkt ist.
    Damit gibt es $M > 0$ mit $\norm {T_n} \le M$ für alle $n ∈ ℕ$, also mit~\eqref{eq:61}
    \[
        \norm{Tx} \xleftarrow[n → \infty ]{} \norm{T_nx } \le M \norm x, ∀ x ∈ X,
    \]
    also die stetigkeit von $T$.
    Jetzt zur Konvergenz:
    Für $\norm x \le$ 1 gilt
    \[
        \norm {T_n x - T_m x } < \epsilon , \quad ∀n, m \ge N_0,
    \]
    also durch Grenzwertbildung $n → \infty $
    \[
        \norm {T_n x - T x } < \epsilon , \quad ∀n \ge N_0,
    \]
    und mit~\eqref{eq:61}
    \[
        \norm {T_n -T} = \sup_{\norm x \le 1} \norm {T_n x - T_x} < \epsilon , \quad ∀ n \ge N_0,
    \]
    das heißt $T_n → T$ wie gewünscht.

    Für den Zusatz haben wir
    \[
        \norm {S(Tx)} \le \norm S \norm {Tx} \le \norm S \norm T \norm x.
    \]
    Da das für alle $x ∈ X$ gilt, haben wir $\norm {ST} \le \norm S \norm T$.
\end{proof}


\begin{korollar}
    Ist $X$ ein Banachraum, dann ist $\L(X) \coloneq \L(X,X)$ eine \emph{Banachalgebra}, das heißt ein vollständiger normierter Vektorraum mit einer Multiplikation, so dass für $T, S ∈ \L(X)$ gilt:
    \[
        \norm {TS} \le \norm T \norm S.
    \]
\end{korollar}

\begin{bemerkung}
    Ist $T ∈ \L(X,Y)$, so ist $\ker T$ als Urbild der abgeschlossenen Menge $\{ 0\}$ stets abgeschlossen in $X$.
    Das Bild hingegen $R(T) \coloneq \im T$ ist im Allgemeinen jedoch nicht abgeschlossen.
    Wann sind Elemente in $\L(X)$ invertierbar?
\end{bemerkung}

\begin{satz}
    Sei $X$ ein Banachraum und $\T ∈ \L(X)$ mit $\limsup\limits_{m → \infty } \norm{T}^{1/m} < 1$. Dann ist $(\id - T)^{-1} ∈ \L(X)$ und es gilt
    \[
        (\id-T)^{-1} = \ lim_{m → \infty } \sum_{n = 0}^m T^n =: \sum_{n = 0}^\infty  T^n ∈ \L(X).
    \]
    mit Konvergenz in $\L(X)$.
\end{satz}
\begin{proof}
    Wähle $m_0$ und $\Theta  < 1$ mit $\norm {T^n} < \Theta ^n$  für $n \ge m_0$.
    Für $S_k = \sum_{n=0}^k T^n$ gilt dann für $m_0 \le k < l$
    \[
        \norm{ S_l - S_k} = \norm { \sum_{n=k+1}^l T^n} \le \sum_{n=k+1}^l \norm{ T^k} \le \sum_{n=k+1}^l \Theta ^n < \epsilon , \quad k, l \ge N_0.
    \]
    Damit ist $(S_k)_{k ∈ ℕ}$ eine Cauchy-Folge in $\L(X)$ und somit konvergent.
    Sei $S$ der Grenzwert. Dann gilt für jedes $x ∈ X$ auch $S_k x \xrightarrow[k → \infty ]{\norm\cdot_{X}} Sx$, also damit ist für alle $x∈ X$
    \[
        (\id - T) Sx = \lim_{k → \infty } (\id -T) S_k x = \lim_{k → \infty } \sum_{n=0}^k (T^n -T^{n-1})x = \lim_{k→\infty } x - T^{k+1}x = x.
    \]
    Damit ist $(\id -T)S = \id$. Da sich analog $S(\id-T) = \id$ auch zeigen lässt, folgt die Behauptung.
\end{proof}


\begin{lemma}
    3.7.6
\end{lemma}
\begin{bemerkung-nn}
    Mit $\Theta  = 1$ geht es nicht immer. Gegenbeispiel: Sei $X = C[0,1] ∩ \{ x(0) =
    0 \}$ und  $M = \{ x ∈ X : g∫_0^1 x(t) dt = 0 \}$.
    Dann ist $M$ ein abgeschlossener linearer Unterraum, weil $T: X → ℝ, ∫_0^1 \cdot$ stetig ist und somit $M = T^{-1}(\{0\})$ als Urbild einer abgeschlossenen Menge in $ℝ$  abgeschlossen ist.
    Angenommen, ($\Theta =1$), es existierte ein $x_\Theta  = x ∈ X$ mit $\norm x_1 = $ und $\norm {x-x_1} \ge 1 $ für alle $x ∈ M$.
    Dann setze
    \[
        c(y) \coloneq \frac{∫_0^1 x_1(t) dt}{∫_0^1 y(t) dt} ∈ ℝ
    \]
    für alle $y \not\in M$. Man beachte, dass dies wohldefiniert ist. 
    Dann ist $x_1 - c(y)y ∈ M$, also $1 \le \norm{ x_1 - c(y)y - x_1} = |c(y)|\norm y$.
    Dann $∫_0^1 x_1 c(y)y\; dt = 0 $ oder $\frac {1}{|c(y)} \le \norm y $ oder $\left| ∫_0^1 y(t)\;dt \right| \le \left| ∫_0^1x_1(t)\;dt \right| \norm y$ für alle $y ∈ X \setminus M)$.
    Wähle $y_n(t) = t^{1/n} ∈ X$, also $\norm {y_n} = 1$.
    Es gilt $\left| ∫_0^1 y_n(t) dt \right| \le \left| ∫_0^1 x_1(t) dt \right| \le 1$ für alle $n ∈ ℕ$, also
    $∫_0^1 x_1(t) dt = 1$ und $x_1(t) \le 1$, was aber bereits impliziert, dass $x_1$ identisch 1 ist. Damit ist $x_1 \not\in X$.
\end{bemerkung-nn}

\begin{satz}
    7.7
\end{satz}
\begin{proof}
    „⇐“ war Korollar 7.4.

    „⇒“. Angenommen, $\dim X = \infty.$ Sei $S^1 \coloneq \{ x ∈ X: \norm x = 1\}$.
    Da $S^1$ abgeschlossen und beschränkt ist, ist $S^1$ nach Annahme kompakt.
    Wähle $x_1 ∈ S^1$ und $M_1 \coloneq \lspan \{ x_1 \} \subsetneq X$.
    $M_1$ ist ein abgeschlossener Unterraum nach Korollar 7.5.
    Nach Ries existiert ein $x_2 ∈ S_1$ mit $\norm {x_2-x_1} \ge \Theta  \coloneq \frac 1 2 $.
    Setze nun $M_2 \coloneq \lspan \{x_1,x_2\}$.
    Da $M_2$ ein abgeschlossener Unterraum ist, existiert ein $x_3 ∈ S_1$ mit $\norm {x_3 - x} \ge \Theta $ für alle $x ∈ M_2$, also insbesondere $\norm {x_3-x_1} \ge \Theta  = \frac 1 2$ und $\norm {x_3-x_2} \ge \Theta  = \frac 1 2$.
    Iterativ (da $\dim X = \infty  $) existiert $x_n ∈ S_1$ mit $\norm {x_m - x_n} \ge \frac 1 2$ für $m \ge n$.
    Somit haben wir eine Folge $(x_n)_{n ∈ ℕ}$ ohne Häufungspunkt in $S^1$ gefunden im Widerspruch zu $S^1$ kompakt.
\end{proof}

Damit sind in unendlich-dimensionalen normierten Räumen weder die Sphären noch die abgeschlossenen Kugeln kompakt.


\begin{definition}
    Ein topologischer linearer Raum $X$ heißt \emph{lokalkompakt}, wenn $0 ∈ X$ eine Umgebung $U$ besitzt, deren Abschluss kompakt ist.
\end{definition}

\begin{korollar}
    Sei $X$ normiert, $\dim X = \infty $. Dann ist $X$ nicht lokalkompakt.
\end{korollar}
\begin{proof}
    Angenommen, dass doch. Dann gibt es $r > 0$, so dass $S_r = \{ x ∈ X : \norm x = r\}\cl U$.
    Da $\cl U$ nach Annahme kompakt ist und $S_r$ abgeschlossen, ist $S_r$ ebenfalls kompakt. Das ist ein Widerspruch.
\end{proof}


\chapter{Unitäre Räume und Hilberträume}
\section{Grundbegriffe}
Sei wieder $\K = \R$ oder $\K = ℂ$.

\begin{definition}
    Sei $X$ ein linearer Raum über $\K$.
    Eine Abbildung $\langle \cdot, \cdot \rangle: X × X → \K$ heißt \emph{Skalarprodukt} auf $X$, falls gilt
    \begin{enumerate}[label=(U\arabic*)]
    \item 
        $\langle x, x \rangle > 0$ für alle $0 \ne x ∈ X$.
    \item
        $\langle  x, y \rangle = \cl {\langle  y, x \rangle}$ für alle $x, y ∈ X$.
    \item
        $\langle x, \alpha y + β z \rangle = \alpha  \langle  x, y \rangle + β \langle  x,z \rangle$ für alle $\alpha , β ∈ \K$, $x,y,z ∈ X$.
    \end{enumerate}
    $(X,\langle -,- \rangle)$ heißt \emph{Skalarproduktraum}, \emph{unitärer Raum} oder \emph{Prähilbertraum}.
\end{definition}

\begin{bemerkung-nn}
    Offenbar ist $\langle -,- \rangle$ in der ersten Komponente konjugiert linear.
\end{bemerkung-nn}

\begin{satz}
    Sei $(X, \langle  -,- \rangle)$ ein unitärer Raum. Dann gelten die folgenden Aussagen:
    \begin{enumerate}
    \item
        Durch $\norm x \coloneq \sqrt{\langle  x, x \rangle}$ wird eine Norm definiert.
        Dadurch wird jeder unitäre Raum auf natürliche Art und Weise normiert und trägt dadurch die induzierte natürliche Topologie.
    \item
        $|\langle  x,y \rangle| \le \norm x \norm y$ mit Gleichheit genau dann, wenn $x$ und $y$ linear abhängig (Cauchy-Schwarz-Ungleichung).
    \item
        $\norm {x+y}^2 + \norm{x-y}^2 = 2(\norm x^2 + \norm y^2)$ (Parallelogrammgleichung),
    \item
        Für $\K = ℝ$ gilt
        \[
            \langle  x,y  \rangle = \frac 1 4 \left( \norm { x+y}^2 - \norm{x-y}^2 \right),
        \]
        für $\K = ℂ$
        \[
            \langle  x, y \rangle = \frac 1 4 \left( \norm {x+y}^2 - \norm{x-y}^2 - i \norm{x+iy}^2 + i\norm{x-iy}^2 \right).
        \]
    \end{enumerate}
\end{satz}
\begin{proof}
    \begin{enumerate}
    \item
        Einfaches Nachrechnen unter Verwendung von (b)
    \item
    Für $y = 0$ ist die Behauptung klar. Sei also $y \ne 0, \alpha  ∈ ℂ$.
    Dann
    \[
        \langle  x + \alpha y, x+\alpha y \rangle = \langle  x, x \rangle + \cl \alpha \langle  y, x \rangle + \alpha  \langle  x,y \rangle + |\alpha |^2 \langle y,y \rangle.
    \]
    Speziell für $\cl \alpha \coloneq - \frac{\langle  x,y \rangle}{\langle y,y \rangle}$ ergibt sich
    \[
        0 \le \langle x + \alpha y, x+\alpha + \rangle = \langle  x,x \rangle - \frac{|\langle x,y \rangle^2|}{\langle y,y \rangle} - \frac{|\langle x,y \rangle^2|}{\langle y,y \rangle} + \frac{|\langle x,y \rangle^2|}{\langle y,y \rangle} = \langle x,x \rangle - \frac{|\langle x,y \rangle^2|}{\langle y,y \rangle}.
    \]
    Durch Umstellen ergibt sich
    \[
        \langle  x, x \rangle \ge \frac{|\langle x,y \rangle|^2}{\langle y,y \rangle} \gdw |\langle x,y \rangle|^2 \le \norm x ^2 \norm y^2.
    \]
    Die CSU erhält man durch Wurzel ziehen.
    Gleichheit gilt genau dann, wenn
    \[
        \langle  x+ \alpha  y, x+\alpha y \rangle = 0 \gdw x + \alpha y = 0,
    \]
    also wenn $x$ und $y$ linear abhängig sind.
    \item
        Es gilt
        \[
            \norm {x \pm y}^2 = \norm x^2 \pm 2\Re(\langle x,y \rangle) + \norm y ^2.
        \]
        Addieren dieser Gleichungen für $+$ und $-$ ergibt die Behauptung.
    \item
        Es gilt
        \begin{align*}
          \norm {x+y}^2 - \norm{x-y}^2 &= (\norm x^2 + 2 \Re \langle x,y\rangle + \norm y^2)  - (\norm x^2 - 2 \Re \langle x,y \rangle + \norm y^2) \\
          & = 4 \Re \langle  x,y \rangle.
        \end{align*}
        Analog haben wir
        \[
            -i \norm{x+iy}^2 + i \norm{x-iy}^2 = … = 4i \Im \langle x,y \rangle,
        \]
        was die Behauptung impliziert.
    \end{enumerate}
\end{proof}

\begin{satz}
    Sei $(X,\norm\cdot)$ ein normierter Raum, der die Parallelogrammgleichung erfüllt.
    Dann definieren
        \[
            \langle  x,y  \rangle = \frac 1 4 \left( \norm { x+y}^2 - \norm{x-y}^2 \right),
        \]
        und
        \[
            \langle  x, y \rangle = \frac 1 4 \left( \norm {x+y}^2 - \norm{x-y}^2 - i \norm{x+iy}^2 + i\norm{x-iy}^2 \right).
        \]
        Skalarprodukte auf $X$ (für $\K = ℝ$ bzw $ℂ$).
\end{satz}
\begin{proof}
    Stupides nachrechnen (oder so ähnlich).
\end{proof}

\begin{bemerkung}
    \begin{enumerate}
    \item
        Die Paralellogrammgleichung ist somit charakteristisch für unitäre Räume.
    \item
        $(C(S),\norm\cdot_\infty )$ mit $S ⊂ ℝ^n$ kompakt erfüllt dies nicht.
    \item
        Die Abbildung $\langle \cdot,\cdot \rangle$ in unitären Räumen ist stetig in beiden Komponenten als unmittelbare Konsequenz aus der Stetigkeit der Norm.
    \end{enumerate}
\end{bemerkung}

\begin{definition}
    Ein bezüglich der Norm $\norm \cdot \coloneq \sqrt{ \langle \cdot,\cdot \rangle}$ vollständiger unitärer Raum $(X,\langle \cdot,\cdot \rangle)$ heißt \emph{Hilbertraum}.
\end{definition}


Hier fehlt eine VL.

\begin{korollar}
    $\hat y$ erfüllt die Gleichung aus dem vorherigen Satz genau dann, wenn $(x- \hat y) \perp Y$ gilt.
\end{korollar}
\begin{proof}
    „⇐“:
    Sei also $\hat y ∈ Y$ mit $x-\hat y \perp Y$, also $x-\hat y \perp (\hat y - y)$ für $y ∈ Y$ beliebig.
    Dann gilt mit Pythagoras
    \[
        \norm{x-y}^2 = \norm{x-\hat y + \hat y - y}^2 = \norm{x-\hat y}^2 + \norm{\hat y - y}^2 \ge \norm{x-\hat y}^2,
    \]
    was die Behauptung impliziert.
\end{proof}
\begin{bemerkung-nn}
    Damit gilt im Hilbertraum das Riesz'sche Lemma (3.7.6) mit $\Theta  = 1$.
    Setze dazu
       $ x_{\Theta =1} \coloneq \frac{x-\hat y }{\norm{x-\hat y}} $
    für ein $x \notin Y$. Dann ist $\norm{x_\Theta } = 1$ und für alle $z ∈ Y$
gilt $\norm {z-x_\Theta }^2 + 2 \Re \langle z,x_\Theta \rangle + \norm{x_\Theta
}^2 \ge 1 = \Theta $.
\end{bemerkung-nn}
\begin{satz}
    Es sei $Y$ ein vollständiger Unterraum eines unitären Raums $X$.
    Dann existiert zu  jedem $x ∈ X$ eine eindeutige Zerlegung der Form
    \[
        x= y + v 
    \]
    mit $y ∈ Y$ und $v ∈ Y^\perp$, das heißt $X = Y \oplus Y^\perp$.
\end{satz}
\begin{proof}
    Jedes $x ∈ X$ lässt sich als $x = \hat y + (x- \hat y)$ schreiben, wobei $\hat y$ wie im Vorherigen Satz ist.
    Dann ist $\hat y ∈ Y$ und $(x-\hat y) ∈ Y^\perp$.
    Für die Eindeutigkeit seien $x = y_1 + v_1 = y_2 + v_2$ zwei Darstellungen von $x$ mit $y_i ∈ Y, v_i ∈ Y^\perp, i=1,2$.
    Dann $y_1 - y_2 = v_2 - v_1$, wobei die linke Seite in $Y$ ist und die rechte in $Y^\perp$, aber $Y ∩ Y^\perp = \{ 0\}$ nach einem vorherigen Resultat, also $y_1 = y_2$ und $v_1 = v_2$.
\end{proof}
\begin{bemerkung-nn}
Weil für jedes $x ∈ X$ das Element $y = \hat y(x) ∈ Y$ in dieser Darstellung eindeutig ist, lässt sich dadurch eine Abbildung $P: X → X, x ↦ y$ definieren.
Diese Abbildung ist eine Projektion, das heißt $P \circ P = P$.
Wir schreiben für $P$ auch $\Proj_Y : X → X$ mit Wertebereich $\im P = Y$ und $P|_Y = \id|_Y$.
\end{bemerkung-nn}
\begin{korollar}
    Falls $M ⊂ X$ ein Unterraum des Hilbertraums $X$ ist, dann gilt
    \[
        \cl M = (M^\perp)^\perp.
    \]
\end{korollar}
\begin{proof}
    „⊂“ wurde bereits in Definition 2.1 gezeigt.

    „$\supset$“: Falls $(M^\perp)\perp \ne \cl M$, dann existiert $x_0 ∈ (M^\perp)^\perp \setminus \cl M$.
    Da $X$ ein Hilbertraum ist, ist  $\cl M$ vollständig.
    Nach dem Satz vom orthogonalen Komplement gibt es eine eindeutige orthogonale Zerlegung von $x_0 = \hat x_0 + h_0^\perp$ mit $\hat x_0 = \Proj_M(x_0) ∈ \cl M$ und $x_0^\perp ∈ (\cl M)^\perp$.
    Da $x_0^\perp ∈ (\cl M)^\perp$, ist auch $x_0^\perp ∈ (M)^\perp$ und $x_0 ∈ (M^\perp)^\perp$, also insbesondere $\langle  x_0, x_0^\perp \rangle = 0$.
    Das bedeute mit Hilfe der Zerlegung
    \[
        0 = \langle  x_0, x_0^\perp \rangle
        = \langle  \hat x_0 + x_0^\perp, x_0^\perp \rangle
        = \langle  \hat x_0, x_0 ^\perp \rangle + \langle x_0^\perp, x_0^\perp \rangle
        = \langle  x_0^\perp, x_0^\perp \rangle
        = \norm{x_0^\perp}^2.
    \]
    somit ist bereits $x_0^\perp = 0$, also $x_0 = \hat x_0 ∈ \cl M$.
    Damit ist $\cl M = (M^\perp)^\perp$.
\end{proof}
\begin{bemerkung-nn}
Die Abbildung $P$ ist beschränkt mit Operatornorm $\norm P = \sup\limits_{x \ne 0} \frac{\norm{P(x)}}{\norm x} \le 1$,
denn für jedes $x = y + v$ mit $y ∈ Y, v ∈ Y^\perp$ gilt
\[
    \norm{P(x)}^2 = \norm{y^2} \le \norm y^2 + 2 \Re \langle  y, v \rangle + \norm{v}^2 = \norm{y +v}^2 = \norm{x}^2.
\]
Desweiteren ist $P$ symmetrisch, das heißt für alle $x_1, x_2 ∈ X $ ist
\[
    \langle P(x_1), x_2 \rangle = \langle x_1, P(x_2) \rangle.
\]
Ist $x_1 = y_1 + v_2$, $x_2 = y_2 + v_2$ mit $y_i ∈ Y, v_i ∈ Y^\perp, i=1,2$, dann ist
\[
    \langle  P(x_1), x_2 \rangle = \langle  y_1,x_2 \rangle = \langle  y_1,y_2 + v_2 \rangle
    = \langle  y_1,y_2 \rangle = \langle  y_1+v_1, y_2 \rangle = \langle x_1, P(x_2) \rangle.
\]
\end{bemerkung-nn}
\begin{korollar}
    Es Sei $Y \ne \{0\}$ ein vollständiger Unterraum des unitären Raums $X$ mit der Projektion $P = \Proj_Y: X → Y ⊂ X$. Dann gilt
    \begin{enumerate}
    \item $x-P(x) \perp Y $ für alle $x ∈ X$.
    \item
        $P$ ist symmetrisch.
    \item
        $P$ ist beschränkt mit Operatornorm $\norm P = 1$.
    \end{enumerate}
\end{korollar}
\begin{proof}
    (1) und (2) wurden bereits gezeigt. Bei (3) fehlt nur noch „$\ge$“.
    Da $P_Y = \id|_Y$ und $Y \ne \{0\}$ ist das aber ebenfalls klar.
\end{proof}
Zentral in der Hilbertraumtheorie ist der Begriff der Hilbertraumbasis.
\begin{definition}
    Ein Orthonormalsystem $(\hat e_k)_{k ∈ ℕ}$ eines unitären Raums $X$ heißt eine Orthonormalbasis oder eine \emph{Hilbertraumbasis}, falls eine der folgenden äquivalenten Bedingungen erfüllt ist:
    \begin{enumerate}
    \item Für alle $x ∈ X$ gilt die Vollständigkeitsrelation
        \[
            \lim_{n → \infty } \norm{x - \sum_{k=1}^n \langle  \hat e_k, x \rangle \hat e_k} = 0 
        \]
    \item
        Für alle $x, y ∈ X$ ist
        \[
            \langle  x,y \rangle = \sum_{k=1}^\infty  \cl{\langle \hat e_k. x \rangle} \langle \hat e_k, y \rangle.
        \]
    \item
        Für alle $x ∈ X$ gilt die Parseval-Gleichung
        \[
            \norm{x}^2 = \sum_{k=1}^\infty  \left| \langle  \hat e_k, x \rangle \right|^2.
        \]
    \end{enumerate}
\end{definition}
\begin{proof}
    Übung.
\end{proof}
\begin{bemerkung-nn}
    \begin{enumerate}
    \item Statt (a) kann man auch
        \[
            x = \lim_{n → \infty } \sum_{k=1}^n \langle  \hat e_k, x \rangle \hat e_k
            = \sum_{k=1}^\infty  \langle  \hat e_k,x \rangle \hat e_k
        \]
        schreiben. Dies nennt man die Fourier-Reihe von $x$.
    \item
        Die approximierenden Elemente
        \[
            \sum_{k=1}^n \langle \hat e_k, x \rangle \hat e_k
        \]
        liegen offenbar in $\lspan S$ wenn $S = \{ \hat e_k : k ∈ ℕ \}$ ist,
        was sich nicht notwendigerweise auf den Grenzwert überträgt.
        Falls $X$ aber vollständig ist (also ein Hilbertraum), so sind diese Aussagen äquivalent zu $\cl{\lspan S}^{\norm\cdot} = X$.
    \end{enumerate}
\end{bemerkung-nn}
\begin{satz}
    \begin{enumerate}
    \item
        Für einen unitärer Raum $X$ gilt: Jede Hilbertraumbasis ist auch ein vollständiges Orthonormalensystem.

    \item
        Ist zusätzlich $X$ ein Hilbertraum und $(\hat e_k)_{k ∈ ℕ}$ ein vollständiges Orthonormalensystem, dann ist $(\hat e_k)_{k ∈ ℕ}$ auch eine Hilbertraumbasis.
    \end{enumerate}
\end{satz}
\begin{proof}
    \begin{enumerate}
    \item
        Sei $S$ wie oben. Sei $x ∈ X$ mit $x \perp S$. Nach (c) gilt dann
        \[
            \sum_{k=1}^\infty  \big| \underbrace{\langle  \hat e_k^\infty , x \rangle}_{=0} \big|^2 = \norm x ^2,
        \]
        also $\norm x = 0$ und $x = 0$.
    \item
        Sei nun $S$ ein abzählbares vollständiges Orthonormalensystem und $X$ ein Hilbertraum.
        Führe den Beweis indirekt.
        Angenommen, $S$ wäre keine Hilbertraumbasis.
        Dann gelten die Eigenschaften (a)-(c) aus der Definition nicht und wegen der obigen Bemerkung ist dann $Y \coloneq \cl{\lspan S} \subsetneq X$.
        $Y$ ist also ein abgeschlossener Unterraum von $X$, und da $X$ Hilbertraum ist, damit vollständig.
        Nach Satz 2.9 ist $X = Y \oplus Y^\perp$.
        Insbesondere ist also $Y^\perp \ne \{ 0\}$.
        Damit gibt es ein $x ∈ X \setminus \{ 0\}$ mit $x ∈ Y^\perp$, also
        \[
            \langle  \hat e_k, x \rangle = 0
        \]
        für alle $k ∈  ℕ$ im Widerspruch zur Vollständigkeit von $S$.
    \end{enumerate}
\end{proof}
\begin{frage-nn}
    Hat jeder Hilbertraum $H$ mit $\dim H = \infty $ ein abzählbares vollständiges ONS (also eine Hilbertbasis)?
\end{frage-nn}
Die Antwort darauf ist nein, aber falls $H$ zusätzlich separabel ist, dann ist sie ja.
Dagegen ist die Existenz eines vollständigen Orthonormalensystems (also eventuell überabzählbar, also keine ONB) kein Problem:
\begin{satz}
    In jedem Hilbertraum $X \ne \{ 0\}$ gibt es ein vollständiges Orthonormalensystem.
    Es lässt sich sogar jedes ONS $S_0$ zu einem vollständigen Orthonormalensystem $\tilde S_0$ mit $S_0 ⊂ \tilde S_0$ ergänzen.
\end{satz}
\begin{proof}
    Simple Anwendung von Zorns Lemma.
\end{proof}
\begin{beispiel}
    \begin{enumerate}
    \item
        Sei $X = L^2(0,2\pi), \K = ℝ$.
        Dann ist ein VONS in $X$ gegeben durch
        \[
            S = \left\{ \frac 1 {\sqrt{2\pi }}\right\}\left\{ \frac 1 {\sqrt{\pi }} \cos(nx) : n ∈ ℕ\right\}\left\{ \frac 1 {\sqrt{\pi }} \sin(nx) : n ∈ ℕ\right\}.
        \]
        In der klassischen Fourieranalysis werden Entwicklungen nach diesem VONS $S$ untersucht.
        Man zeigt dort, dass $\lspan S$ bezüglich $\norm\cdot_\infty $ dicht liegt in $C_{\text{per}}([0,2\pi]) = \{ f: \R\R: f$ ist stetig und $2\pi $-periodisch $\}$.
        Die Aussage von 2.13(2) und (2.10) liefert nur die Begründung
für die Dichtheit von $\lspan S$ in $\norm-_{L^2}$.
    \item
        Durch $(f,g)_\mu  \coloneq ∫_a^b \mu (t) f(t) g(t)\; dt $, wobei $\mu  > 0$ und stetig auf $(a,b)$, ist auf $L^2(a,b)$ ein reelles Skalarprodukt definiert.
        Für verschiedene Gewichtsfunktionen $\mu $ und verschiedene Wahlen von $(a,b)$ erhält man $\mu $-orthogonale Polynomsysteme durch Anwendung des Gram-Schmidt-Verfahrens auf die Monome $\{t^i: i ∈ ℕ_0\}$.
        \begin{enumerate}[label=(\roman*)]
        \item
            $a=-1, b=1$, $\mu (t) = 1$ liefert die Legendre-Polynome.
        \item
            $a=-1, b=1$, $\mu (t) = \frac 1 {\sqrt{1-t^2}}$ liefert die Tschebyscheff-Polynome.
        \item
            $a=0, b=\infty $, $\mu (t) = \exp(-t)$ liefert die Laguerre-Polynome.
        \item
            $a=-\infty , b=\infty $, $\mu (t) = \exp(-t^2)$ liefert die Hermite-Polynome.
        \end{enumerate}
    \item
        Ist $X$ ein unitärer Raum mit ONB, kann er formal vervollständigt werden:
        Sei also $(\hat e_k)_{k ∈ ℕ} ⊂ X$ diese ONB, dann ist
        \[
            H \coloneq \left\{ \sum_{k=1}^\infty  c_k \hat e_k: (c_k)_{k ∈ ℕ} ∈ \ell^2 \right\}
        \]
        ist ein Hilbertraum, den man die Vervollständigung von $X$ nennt.
        Das Skalarprodukt zwischen $x = \sum_{k ∈ ℕ} c_k \hat e_k$  und $y = \sum_{k ∈ ℕ} d_k \hat e_k$
        wird definiert als
        \[
            \langle  x,y \rangle \coloneq \sum_{k=1}^\infty  \cl{c_k} d_k.
        \]
        Tatsächlich kann $H$ mit dem Koordinatenraum $\ell^2 = \ell^2(ℕ)$ identifiert werden.
        Die Abbildung
        \[
            \Phi: \ell^2(ℕ) → H, (c_k)_{k ∈ℕ}\sum_{k=1}^\infty  c_k \hat e_k
        \]
        ist linear, bijektiv und normerhaltend wegen der Parsevalgleichung
        \[
            \norm{x}^2 = \sum_{k=1}^\infty  \left| \langle  \hat e_k, x \rangle \right|^2.
        \]
        Also $\ell^2(ℕ)$ und $H$ isometrisch und insbesondere $H$ vollständig.
    \end{enumerate}
\end{beispiel}


% VL NÄCHSTE WOCHE

Der Satz 4.1 liefert also, dass die Abbildung $J_x: X → X', y ↦ y'$ definiert
durch $y': X → \K, x ↦ \langle  y,x \rangle$ bijektiv ist.
Wir schreiben nun
\[
\lAngle J_x(y),x  \rAngle = \lAngle J_x(y),x \rAngle_{X'×X} \coloneq J(x)(y)[x]
= \langle  y,x \rangle.
\]
Diese Abbildung ist sesquiliniear, das heißt
\[
    J_x (y_1 + y_2) = J_x (y_1) + J_x(y_2), \quad y_1, y_2 ∈ X,
\]
\[
    J_x(\alpha y) = \cl{\alpha} J_x(y), \quad \alpha\K,
\]
denn
\[
    \lAngle J_x(\alpha y),x \rAngle = \langle  \alpha y, x \rangle = \cl \alpha \langle  y, x \rangle = \cl \alpha J_x(y) [x] = \cl \alpha \lAngle  J_x(y), x \rAngle
    \lAngle \cl \alpha J_x(y), x \rAngle,
\]
also $X \cong X'$ sesquilinear isomorph.

Gilt da sauch topologisch?
Die Topologie von $X'$ sei hierbei die von $\L(X, \K)$, also die von der Norm $\norm{y'}_{X',N} = \sup_{\norm{x} \le 1}|y'[x]|$ erzeugte.
\begin{satz}
    $X$ und $X'$ sind Hilberträume und  $J_x: X → X'$ ist kanonischer sesquilinearer Isomorphismus, der die Norm erhält, also eine Isometrie.

    Genauer gilt:
    \begin{enumerate}
    \item
        $\langle  y_1', y_2' \rangle_{X'} \coloneq \cl{ \langle y_1, y_2 \rangle_X}$, wobei $J_x(y_1) = y_1', J_x(y_2) = y_2'$, macht $X'$ zum Skalarproduktraum.
    \item
        Die durch $\langle  -,- \rangle_{X'}$  induzierte Norm
        \[
            \norm{y'}_{X',S} = \sqrt{\langle y', y' \rangle_{X'}}
        \]
        ist gerade die von $X' = \L(X, \K)$ bekannte, das heißt, $\norm{y'}_{X',S} = \norm{y'}_{X',N}$.
    \item
        Da $(X',\norm-_{X',N})$ schon bekanntlich vollständndig ist, ist $(X', \langle -,- \rangle)$ damit ein Hilbertraum.
    \item
        $J_x: X → X'$ ist eine Isometrie.
    \end{enumerate}
\end{satz}
\begin{proof}
    \begin{enumerate}
    \item
        Beispielsweise ist
        \[
            \langle \alpha  y_1' , y_2' \rangle_{X'} \stackrel{\text{def}}{=} \cl{\langle \cl \alpha y_1,  y_2 \rangle_X} = \cl{ \alpha \langle  y_1, y_2 \rangle_X} = \cl{\alpha} \langle y_1',y_2' \rangle_{X'},
        \]
        die anderen Eigenschaften folgen analog.
    \item 
        Wegen $y'[x] = \langle  y,x \rangle$ und $\norm{y'}_{X',S} = \sqrt{\langle  y', y' \rangle_{X'}} = \sqrt{\langle  y, y \rangle_X} = \norm{y}$, das heißt, es genügt, zu zeigen, dass
        \[
            \norm{y'}_{X',N} = \sup_{\norm x \le 1} |y'[x]| = \norm{y}_{X} \quad \text{für alle $y ∈ X$}.
        \]
        hierbei ist aber „$\le$“ gerade die Cauchy"=Schwarzsche Ungleichung, für „$\ge$“ wähle $x = \frac y {\norm y _{X}}$ für $y \ne 0$ ($y=0$ ist sowieso klar).
    \item
        nichts zu zeigen.
    \item
        $J_x: X → X'$ ist eine Isometrie, denn $y  ↦ J_x(y) = y'$ und $\norm{J_X(y)}_X = \norm{y'}_{X'} = \norm{y}_X$ für alle $y ∈ X$.
    \end{enumerate}
\end{proof}

\section{Separable Hilberträume}
\begin{definition}
    Ein metrischer Raum $(X,d)$ heißt \emph{separabel}, wenn es $U ⊂ X$ dicht
    und abzählbar gibt.
\end{definition}

\begin{beispiele}
    $ℝ^n, ℂ^n, \ell^2, L^2(\Omega)$ für $\Omega ⊂ ℝ^n$ offen sind separable Hilberträume.
\end{beispiele}

\begin{satz}
    In einem separablen unendlich"=dimensionalen Hilbertraum $(X,\langle -,- \rangle)$ gilt
    \begin{enumerate}
    \item Jedes ONS in $X$ ist höchstens abzählbar.
    \item
        Sei $S = (\hat e_i)_{i ∈ ℕ}$ ein VONS in $X$. Dann existiert zu jeder
        Folge $\alpha  = (\alpha _k)_{k ∈ ℕ}\ell^2$ genau ein $x ∈ X$ mit $\langle  \hat
        e_k, x \rangle = \alpha _k, k ∈ ℕ$ (Satz von \emph{Riesz-Fischer}).
    \item
        $X$ ist isometrisch isomorph zum $\ell^2$. Insbesondere sind
        $L^2(\Omega)$ und $\ell^2$ isometrisch isomorph.
    \end{enumerate}
\end{satz}


\section{Riesz'scher Darstellungssatz und Lax-Milgram}
Für einen topologischen linearen Raum $X$ ist der Dualraum $X' = \{x': X → \K, x' $ linear und stetig $\}$ definiert.
Im Allgemeinen kann auch $X' = \{0\}$ gelten.
Ist $X$ jedoch ein Hilbertraum, so ist stets $X' \ne \{0\}$, denn zu $y ∈ X$ ist durch $y'[x] \coloneq \langle y,x \rangle, x ∈ X$  jeweils ein $y' ∈ X'$ erklärt.
Tatsächlich bekommt man dadurch sogar schon alle Elemente des Dualraums:


\chapter{Der Satz von Hahn-Banach und seine Konsequenzen}
\section{Fortsetzbarkeit linearer Funktionale}

Wir fragen uns, ob sich Abbildungen so erweitern, dass gewisse Eigenschaften (wie z.B. Linearität oder Stetigkeit) erhalten bleiben.

\begin{definition}
    Eine Abbildung $A: M → Y$ heißt eine Fortsetzung einer Abbildung $A_0: M_0 → X$, falls
    \begin{enumerate}
    \item $ M_0 ⊂ M$,
    \item $∀x ∈ M_0: A_0 x = Ax $.
    \end{enumerate}
    Wir schreiben dann $A = A|_{M_0}$.
\end{definition}

\begin{satz}
    Seien $(X,\norm-)$ und $(X_0,\norm-)$ normietre Räume, $X_0 ⊂ X$ dicht in $X$.
    Weiter sei $(Y, \norm-_{Y})$ ein Banachraum und $A_0 : X_0 → Y$ stetig und linear.
    Dann gibt es genau eine stetige lineare Fortsetzung $A : X → Y$ von $A_0$ auf $X$.
    Für diese gilt:
    \[
        \norm{A_0}_{\L(X_,Y)} = \norm{A}_{\L(X,Y)}.
    \]
\end{satz}
\begin{proof}
    Zeigen wir zunächst die Existenz der Fortsetzung.
    Da $X_0$ dicht in $X$ ist, existiert zu jedem $x ∈ X$ eine Folge $(x_n)_{n \ge1}$, die ganz in $X_0$ liegt und gegen $x$ konvergiert.
    Wir behaupten, dass $(A_0x_n)_{n ∈ ℕ}$ eine Cauchy-Folge in $Y$ ist.
    Dazu beachte, dass
    \[
        \norm{A_0 x_n - A_0 x_m}_{Y} \le \norm{A_0}_{\L(X_0,Y)} \norm{x_n-x_m} \xrightarrow[n,m → \infty ]{} 0.
    \]
    Da $Y$ ein Banachraum ist, ist $(A_0x_n)_{n\ge1}$ konvergiert, etwa gegen $y$.
    Wir setzen $Ax \coloneq y$.
    Zunächst ist $A$ wohldefiniert, denn wenn $(z_n)_{n \ge 1}$ eine weitere Folge mit $\lim_{n → \infty } z_n = x$ ist, dann gilt
    $z_n - x_n \xrightarrow[n→\infty ]{} 0$ und
    \begin{align*}
        \norm{A_0 z_n - y} &\le \norm{A_0 z_n - A_0 x_n} + \norm{A_0 x_n - y} \\
        & \le  
        \norm{A_0} \norm{z_n - x_n} + \norm{A_0 x_n - y} \xrightarrow[n→\infty ]{} 0.
    \end{align*}
    Offensichtlich ist $A$ eine Fortsetzung von $A_0$.
    Dass $A$ linear ist, ist ebenfalls klar.
    Zur Stetigkeit ist
    \begin{align*}
        \norm{Ax}_Y &= \norm{\lim_{n → \infty } A_0 x_n}_Y = \lim_{n → \infty } \norm{A_0 x_n}_{Y} \\
        &\le
        \lim_{n → \infty } \norm{A_0}_{\L(X_0,Y)} \norm{x_n}_X = \norm{A_0} \norm{x}.
    \end{align*}
    Damit ist $A$ beschränkt, also auch stetig.

    Es gilt $\norm{A_0}_{\L(X_0,Y)} = \norm{A}_{\L(X,Y)}$:
    „$\ge$“ ist aus dem Vorherigen klar. Für die andere Ungleichung ist
    \[
        \norm{A}_{L(X,Y)} =
        \sup_{\norm{x \le 1}, x ∈ X} \norm{Ax}_{Y}
        \ge
        \sup_{\norm{x \le 1}, x ∈ X_0} \norm{Ax}_{Y} = \norm{A_0}_{\L(X_0,Y)}.
    \]

    Für die Eindeutigkeit sei $B: X → Y$ eine weitere stetige, lineare Fortsetzung von $A_0$.
    Wie oben existiert zu jedem $x ∈ X$ eine Folge $(x_n)_{n ∈ ℕ} ⊂ X$ mit $\lim_{n → \infty } x_n = x$.
    Dann ist
    \[
        Ax_n = A_0 x_n = Bx_n \quad ∀ n ∈ ℕ
    \]
    und für $x ∈ X$
    \[
        \norm{B_x - A_x} \le \norm{B_x - Bx_n} + \norm{Bx_n - Ax_n} + \norm{Ax_n - Ax} \xrightarrow[n→\infty ]{} 0,
    \]
    da $A$ und $B$ stetig sind. Also $Bx = Ax$ für alle $x ∈ X$ und damit $B = A$.
\end{proof}

\begin{korollar}
    Ist $A ∈ \L(X,Y)$, $X, Y$ normiert sowie $Y$ vollständig und $M ⊂ X$ dicht, dann gilt:
    Falls $Ax = 0$ für alle $x ∈ M$, dann ist $A$ schon die Nullabbildung auf $X$.
\end{korollar}
\begin{proof}
    ~
\end{proof}

Ist $X_0$ nicht dicht in $X$, wird die Fortsetzung schwieriger.


\begin{satz}
    Auf dem linearen Raum $X$ über $ℝ$ gebe es eine Abbildung $p: X → ℝ$ mit:
    \begin{enumerate}[label=(\roman*)]
    \item
        $p(\alpha x) = \alpha p(x)$ für alle $\alpha  \ge 0, x ∈ X$ (positiv homogen)
    \item
        $p(x+y) \le p(x) + p(y)$ für alle $x, y ∈ X$ (subadditiv)
    \end{enumerate}

    Weiter seine $X_0$ ein linearer Teilraum von $X$ und $f_0 : X_0 → ℝ$ eine lineare Abbildung mit
    \[
        ∀x ∈ X_0 : f_0(x) \le p(x).
    \]
    Dann gibt es eine lineare Fortsetzung $f: X → ℝ$ von $f_0$, welche die Ungleichung respektiert, das heißt
    \[
        f|_{X_0} = f_0 \quad \text{und} \quad ∀x ∈ X: f(x) \le p(x).
    \]
\end{satz}
\begin{bemerkung-nn}
    Halbnormen  oder Normen $p$ Erfüllen die Voraussetzungen dieses Satzes.
\end{bemerkung-nn}
\begin{proof}
    Schritt 1.
    Wir setzen $f_0$ auf $X_1 \coloneq X_0 \oplus \lspan{x_1}$  für ein $x_1 \not\in X$ (existiert immer solange $X_0 \subsetneqq X$).
    Offenbar hat jedes $x ∈X_1$ eine eindeutig Darstellung als
    $ y = y + \alpha x_1 $,  mit $y ∈ X_0$, $\alpha  ∈ ℝ$.
    Dann ist mit $c ∈ ℝ$ beliebig
    \[
        f(x) = f(y + \alpha (x_1)) \coloneq f_0(y) + \alpha c
    \]
    eine lineare Abbildung $X_1 → ℝ$, die $f_0$ fortsetzt.
    Wir müssen $c$ so wählen, dass $f(x) \le p(x)$ für alle $x ∈ X_1$, also $f_0(y) + \alpha c \le p(y+\alpha x_1)$ für alle $y ∈ X_0, \alpha  ∈ ℝ$.
    Mit (i) ist diese Bedingung äquivalent zu zwei anderen Bedingungen:
    \begin{enumerate}
    \item
        Für $a > 0$: $f_0(y/\alpha ) + c \le p(y/\alpha  + x_1)$.
    \item
        Für $\alpha  < 0$: $f_0(-y/\alpha ) - c \le p(-y/\alpha  - x_1)$ 
    \end{enumerate}
    für alle $y ∈ X_0$. Der Fall $\alpha  = 0$ ist nach Annahme ohnehin klar.
    Um diese Bedingungen erfüllen zu können, muss $c ∈ ℝ$ so gewählt werden, dass
    \[
        ∀y_1, y_2 ∈ X_0: f_0(y_1) - p(y_1 - x_1) \le c \le p(y_2 + x_2) - f_0(y_2).
    \]
    Das ist möglich, da
    \[
        f_0(y_1) + f_0(y_2) = f_0(y_1+y_2) \le p(y_1 + y_2) = p(y_1 - x_1 + y_2 + x_1) \le p(y_1 - x_1)+p(y_2+x_1).
    \]
    Folglich gilt
    \[
        \sup_{y_1 ∈ X_0} f_0(y_1-p(y_1-x_1)) \le \inf{y_2 ∈ X_0} p(y_2+x_1)-f_0(y_2).
    \]


    Schritt 2.
    Finde eine maximale Fortsetzung mit dem Lemma von Zorn.
    Betrachte dazu
    \[
        \{: X \supset D_g \supset X_0 → ℝ\}: g|_{X_0} = f_0 ∧ ∀x ∈ D_g: g(x) \le p(x) \}.
    \]
    Diese Menge ordnen wir mit $\succeq$ definiert durch
    \[
        h \succeq g \gdw h \text{ ist Fortsetzung von $g$}.
    \]
    Nach dem Lemma von Zorn existiert eine maximale Fortsetzung $g^*$ von $f_0$ mit $g^*(x) \le p(x)$ für alle $x ∈ X$.
    Wäre $D_{g^*}$ nicht $X$, so verfahre wie in Schritt 1 im Widerspruch zur Maximalität.
    Damit hat $g^*$ die gewünschten Eigenschaften.
\end{proof}

\begin{bemerkung-nn}
    \begin{enumerate}
    \item 
    Ohne die Zusatzforderung $f(x) \le p(x)$ für alle $x ∈X$ ist die lineare Fortsetzbarkeit trivial.
    \item 
        Eine Fortsetzung für lineare Funktionale $f_0: X_0 → \K = ℂ$ ist analog möglich. % yos IV 4
    \end{enumerate}
\end{bemerkung-nn}

%% HIER FEHLT EINE VORLESUNG

\begin{satz}[5.3.1]
    Sei $(X,\norm\cdot)$ ein normierter Raum über $ℝ$, $M ⊂ X$ abgeschlossen und konvex und $0 ∈ M$.

    Dann existiert zu jedem $x_0 \not\in M$ ein $f ∈ X'$ mit
    \[
        f(x_0) > 1 ∧ ∀ x ∈ M: f(x) \le 1.
    \]
\end{satz}
Die Hyperebene $H = \{ x ∈ X: f(x) = 1 + \epsilon \}$ für $0 < \epsilon  < f(x_0) < 1$ trennt also $x_0$ und $M$.

\begin{proof}
    Setze $2r \coloneq \inf_{y ∈ M} \norm{y - x_0}$  (positiv, da $M$ abgeschlossen).
    Sei $N \coloneq \cl{M + \cl{B_r(0)}} = \cl{\{ z = y + u: y ∈ M, u ∈ \cl{B_r(0)}\}} ⊂ X$.
    Dann ist (i) $N$ abgeschlossen und (ii) $\cl{B_r(0)} ⊂ N$, da $0 ∈ M$, insbesondere ist $0 ∈ N^\circ$.
    (iii) ist $N$ konvex: Es genügt, zu zeigen, dass $A = M + B_r(0)$ konvex ist, denn dann ist auch $\cl A$ konvex.
    Sei $x _i = y_i + v_i, y_i ∈ M, v_i ∈ \cl{B_r(0)}, i=1,2$ und $\alpha  ∈ (0,1)$. Dann ist
    \[
        \alpha x_1 + (1-\alpha )x_2 = \underbrace{[\alpha y_1 + (1-\alpha )y_2]}_{∈ M} + \underbrace{[\alpha u_1+(1-\alpha )v_2]}_{\cl{B_r(0)}}.
    \]
    (iv) ist $x_0 \not\in N$.
    Angenommen, $x_0 ∈ N$. Dann existiert eine Folge $z_n = y_n + u_n$ in $A$ mit $z_n → x_0 (n→\infty )$.
    Dann ist für $n_0$ hinreichend groß
    \[
        \frac r 2 > \norm{z_{n_0} - x_0} = \norm{y_{n_0 - x_0} + u_{n_0}} \ge |\underbrace{\norm{y_{n_0-x_0}}}_{\ge 2r} - \underbrace{\norm{u_{n_0}}}_{\le r}| \ge r.
    \]

    Verwende nun das Minkowski-Funktional
    \[
        p_N(x) \coloneq \inf \{ρ > 0: ρ^{-1} x ∈ N\}, \quad x ∈ X.
    \]
    Dieses hat die Eigenschaften
    \begin{enumerate}
    \item
        $p_N(\alpha x) = \alpha p_n(x),\quad \alpha  \ge 0, x ∈ X$ (positiv homogen)
    \item
        $p_N(x+y) \le p_N(x) + p_N(y), \quad x, y ∈ X$ (subadditiv)
    \item
        $p_N(x) \le 1 \iff x ∈ N$
    \item
        Ist zusätzlich $\cl{B_r(0)} ⊂ N$, so gilt $p_nNx) \le r^{-1}\norm x$ für alle $x ∈ X$.
    \end{enumerate}
    Sei nun $X_0 \coloneq \lspan\{x_0\}$ und $f_0 : X_0 → ℝ$ linear definiert durch $f_0(x_0) \coloneq p_N(x_0)$.
    Wir behauptung, dass $f_0 (x) \le p_N(x)$ für alle $x  = \lambda x_0 ∈ X_0$.
    Falls $\lambda  \ge 0$, so ist $f_0(x) = f_0(\lambda x_0) = \lambda p_N(x_0) = p_N(\lambda x_0) = p_N(x)$.
    Falls $\lambda  < 0$, so ist wegen $p_n \ge 0$ ohnehin $f_0(\lambda x_0) = \lambda p_N(x_0) \le 0 \le p_N(\lambda x_0)$.
    Da $p_N$ die Bedingungen (i) und (ii) aus Hahn-Banach erfüllt,
    gibt es eine lineare Fortsetzung $f$ von $f_0$ mit $f(x) \le p_N(x)$ für alle $x ∈ X$.

    Nun ist $f$ stetig, also $f ∈ X'$, denn für alle $x ∈ X$ gilt
    \begin{multline*}
        |f(x) = \max\{f(x), -f(x)\}  = \max\{f(x),f(-x)\} \le \max\{p_N(x),p_N(-x)\}  \\
        \le \max\left\{\frac{\norm{x}}{r},\frac{\norm{-x}}{r}\right\} = \frac{\norm x}{r}.
    \end{multline*}

    Außerdem erfüllt $f$ die Gleichung 3.1 (?), denn
    \[
        f(x_0) = f_0(x_0) = p_n(x_0) > 1
    \]
    und für $x ∈ M ⊂ N$ gilt
    \[
        f(x) \le p_N(x) \le 1.
    \]
\end{proof}

\section{Einbettung von $X$ in seinen Bidualraum}
Zunächst zur Motivation: Sei $X$ ein normierter linearer Raum.
Dann existiert $X'$ und ist ein Banachraum.
Aber dann existiert auch $X'' \coloneq (X')'$ und ist ebenfalls ein Banachraum.
Unser Ziel wird es nun sein, $X$ in $X''$ einzubetten.

\begin{definition}
    Die kanonische Abbildung $J_0: X → X''$ ist definiert durch
    \[
        J_0(x) [x'] = \lAngle J_0(x), x' \rAngle_{X''×X'} \coloneq \lAngle x', x \rAngle_{X'×X} = x'[x]\K
    \]
    für $x ∈ X, x' ∈ X'$.

    Offensichtlich gilt für $x ∈ X$ fest $J_0(x): X' → \K$ linear, aber $J_0(x)$ ist auch stetig bzw beschränkt:
    Dazu ist
    \[
        |J_0(x)[x']| = | \langle  \langle x',x \rAngle \le \norm{x'}_{X'} \underbrace{\norm{x}_X}_{=: M}.
    \]
    Also ist $J_0(x) ∈ X''$, also insbesondere $J_0$ wohldefiniert.
    Wegen der linearität von $J_0$ in $x$ schreiben wir statt $J_0(x)$ auch $J_0 x$.
\end{definition}

\begin{satz}
    Die kanonische Abbildung $J_0: X → X''$ ist eine normerhaltende lineare Einbettung von $X$ in seinen Bidualraum $X''$.
\end{satz}

\begin{warnung-nn}
    $J_0$ ist in der Regel nicht surjektiv.
\end{warnung-nn}

\begin{proof}
    Zur Injektivität: Seien $x_1, x_2 ∈ X$ mit $J_0x_1 = J_0x_2$.
    Dann ist für jedes $x' ∈ X'$
    \[
        \lAngle x',x_1 \rAngle = J_0 x_1[x'] = J_0x_2[x'] = \lAngle x', x_2 \rAngle,
    \]
    also wegen Linearität von $x'$
    \[
        \lAngle x', x_1-x_2 \rAngle = 0.
    \]
    Mit Folgerung 2.3(1) folgt $x_1-x_2 = 0$.

    Zur Isometrieeigenschaft bleibt zu zeigen: $\norm{J_0x} = \norm{x}$ für alle $x ∈ X''$.
    „$\le$“: Aus (4.1) folgt bereits
    \[
        \norm{J_0(x)}_{X''} = \sup_{\norm{x'} \le 1} |J_0(x)[x'] \le \norm{x}_X.
    \]
    „$\ge$“: Zu $x_0 ∈ X$ existiert nach Korollar 2.1 ein $x_0' ∈ X'$ mit
    $\norm{x_0'}_{X'} = 1$ und $x_0'[x_0]= \norm{x_0}$.
    Also folgt
    \[
        \underbrace{|J_0x_0[x_0']|}_{\le \norm{J_0x_0}_{X''}} = \lAngle  x_0', x_0 \rAngle = \norm{x_0}.
    \]
    Da $x_0$ beliebig war, gilt $\norm{J_0x}_{X''} \ge \norm{x}$.
\end{proof}

\begin{definition}
    Ein Banachraum $X$ heißt \emph{reflexiv}, wenn $J_0$ surjektiv ist, also $X$ und $X''$ isomorph sind vermöge $J_0$.
\end{definition}

\begin{bemerkung-nn}
    Ein unvollständiger normierter Raum hätte offensichtlich keine Chance, reflexiv zu sein.
\end{bemerkung-nn}

\begin{warnung-nn}
    „vermöge $J_0$“ in der Definition ist wesentlich, denn es gibt Beispiele mit $X \cong X''$, aber $J_0$ ist nicht surjektiv. %% werner, I 4.7
\end{warnung-nn}

\begin{satz}
    Jeder Hilbertraum $H$ ist reflexiv
\end{satz}
\begin{proof}
    Übung.
\end{proof}

\begin{bemerkung-nn}
    Offensichtlich sind $H$ und $H''$ isometrisch isomorph:
    Denn $H$ und $H'$ sind bereits konjugiert linear isomorph via $J_H, X → X'$ (Kapitel IV, \S 5, aus Ries'schem Darstellungssatz).
    Mit dem gleichen Argument sind $H'$ und $H''$ konjugiert linear isomorph via $J_{H'}$, also $H$ und $H''$ linear isometrisch durch $J_{H'} \circ J_H$.
    Dies genügt aber nicht für den Nachweis der Reflexivität.
    Dafür müssen wir zu $x'' ∈ H''$ ein $x ∈ H$ finden mit $J_0x = x''$.
\end{bemerkung-nn}

\begin{bemerkung-nn}
    Wozu Reflexivität gut ist, werden wir später im Kapitel über schwache Topologien genauer sehen.
    Beispielsweise ist $\cl{B_1(0)}$ im reflexiven Banachraum $X$ schwach folgenkompakt, das heißt jede Folge in $\cl{B_1(0)}$ hat eine schwach konvergente Teilfolge mit Grenzwert in $\cl{B_1(0)}$.
    Dies ist zum Beispiel in der Variationsrechnung sehr wichtig.
\end{bemerkung-nn}

\begin{definition}
    Eine Folge  $(x_n)_{n ∈ ℕ}$ in einem normierten Raum $X$ heißt \emph{schwach konvergent} gegen $x ∈ X$ (in Zeichen: $x_n \xrightharpoonup[n → \infty ]{} x$), wenn
    \[
        \lim_{n → \infty } x'[x_n] = x'[x]
    \]
    für alle $x' ∈ X'$ gilt.
\end{definition}

\begin{bemerkung-nn}
    Der Grenzwert (so er denn existiert) ist eindeutig. Denn ist $x'[x] = x'[\tilde x]$ für alle $x' ∈ X'$, so folgt $x = \tilde x$ mit Folgerung 2.3 (2).
\end{bemerkung-nn}

\begin{beispiel-nn}
    Für $(\hat e_i)_{i ∈ ℕ}$ Hilbertraumbasis in einem separablem Hilbertraum $X$ gilt
    \[
        \hat e_i \rightharpoonup 0 ∈ X (i → \infty )
    \]
\end{beispiel-nn}

\begin{bemerkung-nn}
    $(\hat e_i)_{i ∈ ℕ}$ ist nicht konvergent in der Normtopologie, die Folge ist noch nicht mal Cauchy, insbesondere ist $\norm{\hat e_i - 0} \not\rightarrow 0 (i → \infty )$.
\end{bemerkung-nn}

\begin{proof}
    Der kanonische Isomorphismus $J_X: X  → X', y ↦ y'$ mit $y'[x] = \langle y,x \rangle$ für alle $x ∈ X$ liefert
    \[
        X' = \{ x' : x' ∈ X'\} = \{ J_X(y) : y ∈ X\}.
    \]
    Zu zeigen ist $\lim\limits_{i → \infty }x'[\hat e_i] = x'[0]$ für alle $x' ∈ X'$, also äquivalent
    $\lim\limits_{i → \infty } J_x(y)[\hat e_i] = J_x(y)[0]$ für alle $y ∈ X$ bzw. $\lim\limits_{i → \infty } \langle y, \hat e_i \rangle = \langle y, 0 \rangle$ für alle $y ∈ X$.

    Sei also $y ∈ X$ fest gewählt. Dann ist $y = \sum_{i=1}^\infty \alpha _i \hat e_i$ mit $\alpha _i = \langle  \hat e_i, y \rangle$.
    Es gilt $\sum_{i=1}^\infty  |\alpha _i|^2 < \infty $ (vgl Def 4.2.12).
    Damit folgt $\alpha _i = \langle  \hat e_i, y \rangle0 (i → \infty )$, weil $\alpha\ell^2$.
    Damit folgt die Schwache Konvergenz von $(\hat e_i)_{i ∈ ℕ}$.
\end{proof}


\begin{satz}
    Sei $M$ ein abgeschlossener Unterraum eines Banachraums $(X, \norm -)$.
    \begin{enumerate}
    \item
        Ist $X$ reflexiv, so ist auch $(M, \norm -)$ reflexiv.
    \item
        Ist $X$ ist reflexiv, so auch $X'$.
    \end{enumerate}

\end{satz}


%%% Local Variables:
%%% mode: latex
%%% TeX-master: "funkana-ebook"
%%% End: